The Flat Earth Society

Flat Earth Discussion Boards => Flat Earth Theory => Topic started by: devils advocate on September 20, 2017, 10:49:53 AM

Title: Why is the Earth accelerating at9.8 metres per second?
Post by: devils advocate on September 20, 2017, 10:49:53 AM
I have read the Wiki and FAQ; (UA asserts that the Earth is accelerating 'upward' at a constant rate of 9.8m/s^2)

But am struggling to find either a reason why or proof that the earth IS accelerating at 9.8 metres per second:

Wiki states: "The are several explanations for UA. As it is difficult for proponents of Flat Earth Theory to obtain grant money for scientific research, it is nigh on impossible to determine which of these theories is correct."

I was hoping that a FE could explain why this theory exists without this reasoning and evidence? Why does it make sense to you and how did this exact speed come about? As for the several explanations for UA; what are they please as I could not see them listed anywhere?

many thanks
Title: Re: Why is the Earth accelerating at9.8 metres per second?
Post by: 3DGeek on September 21, 2017, 04:00:05 AM
I have read the Wiki and FAQ; (UA asserts that the Earth is accelerating 'upward' at a constant rate of 9.8m/s^2)

But am struggling to find either a reason why or proof that the earth IS accelerating at 9.8 metres per second:

Wiki states: "The are several explanations for UA. As it is difficult for proponents of Flat Earth Theory to obtain grant money for scientific research, it is nigh on impossible to determine which of these theories is correct."

I was hoping that a FE could explain why this theory exists without this reasoning and evidence? Why does it make sense to you and how did this exact speed come about? As for the several explanations for UA; what are they please as I could not see them listed anywhere?

many thanks

I confess I'm a bit puzzled as to why FE'ers do this.

If the Earth is an infinite disk, of decent thickness - then regular "per Isaac Newton" gravity is a reasonable explanation.


Title: Re: Why is the Earth accelerating at 9.8 metres per second?
Post by: devils advocate on September 21, 2017, 07:23:48 AM
Indeed. Does this also mean that when an aeroplane is flying apparently at a stable 30,000 feet it is in actual fact in ascent as surely if it was travelling on a level plain it would be hit by the upwards earth?

This particular idea needs a lot more work FE.
Title: Re: Why is the Earth accelerating at 9.8 metres per second?
Post by: juner on September 21, 2017, 09:31:54 AM
... as surely if it was travelling on a level plain it would be hit by the upwards earth?

No, that is not how it works at all...
Title: Re: Why is the Earth accelerating at 9.8 metres per second?
Post by: devils advocate on September 21, 2017, 10:28:47 AM
... as surely if it was travelling on a level plain it would be hit by the upwards earth?

No, that is not how it works at all...

I gathered Junker..........I will have to wait to see if any alternative explanations are offered in this thread as the Wiki or FAQ does not explain enough.

Tom's empirical POV states that if someone jumps out of the plane they do not fall down to earth, instead the earth rushes up to meet them..........hmmmmmmmmmmm
Title: Re: Why is the Earth accelerating at 9.8 metres per second?
Post by: juner on September 21, 2017, 01:00:50 PM
I gathered Junker..........I will have to wait to see if any alternative explanations are offered in this thread as the Wiki or FAQ does not explain enough.

It is no different than RE in this regard (in terms of the concepts involved). The Universal Acceleration model has everything in earth's non-inertial frame of reference accelerating at the same rate. Due to the Equivalence Principle, this is literally no different than "gravity."
Title: Re: Why is the Earth accelerating at 9.8 metres per second?
Post by: Pete Svarrior on September 21, 2017, 01:01:38 PM
Indeed. Does this also mean that when an aeroplane is flying apparently at a stable 30,000 feet it is in actual fact in ascent as surely if it was travelling on a level plain it would be hit by the upwards earth?
[...]
I gathered Junker..........I will have to wait to see if any alternative explanations are offered in this thread as the Wiki or FAQ does not explain enough.
This is explained both in our Wiki (https://wiki.tfes.org/Universal_Acceleration#Equivalence_Principle) and on Wikipedia (https://en.wikipedia.org/wiki/Equivalence_principle). In short: generating enough lift to avoid a flat plane accelerating at 9.8%5Cfrac%7Bm%7D%7Bs%5E2%7D is equivalent and indistinguishable from generating enough lift to prevent an aeroplane from crashing down due to the 9.8%5Cfrac%7Bm%7D%7Bs%5E2%7D acceleration caused by RET's gravitation.

If the Earth is an infinite disk, of decent thickness - then regular "per Isaac Newton" gravity is a reasonable explanation.
Indeed, and many infinite plane FE'ers have advocated for that as the answer.
Title: Re: Why is the Earth accelerating at 9.8 metres per second?
Post by: devils advocate on September 21, 2017, 01:51:07 PM
Indeed. Does this also mean that when an aeroplane is flying apparently at a stable 30,000 feet it is in actual fact in ascent as surely if it was travelling on a level plain it would be hit by the upwards earth?
[...]
I gathered Junker..........I will have to wait to see if any alternative explanations are offered in this thread as the Wiki or FAQ does not explain enough.
This is explained both in our Wiki (https://wiki.tfes.org/Universal_Acceleration#Equivalence_Principle) and on Wikipedia (https://en.wikipedia.org/wiki/Equivalence_principle). In short: generating enough lift to avoid a flat plane accelerating at 9.8%5Cfrac%7Bm%7D%7Bs%5E2%7D is equivalent and indistinguishable from generating enough lift to prevent an aeroplane from crashing down due to the 9.8%5Cfrac%7Bm%7D%7Bs%5E2%7D acceleration caused by RET's gravitation.

Thanks for responding, I don't follow that they are the same. On RE the ground remains at the same level whilst the plane flies above it, generating thrust to avoid the pull of gravity. On FE the plane is still in the air but crucially the ground is NOT static, it is rushing up towards the plane.

A trampoline jump: RE- a certain force is applied by the jumper to raise themselves 10 feet above the trampoline, then they fall down 10 feet back to trampoline

FE - The trampoline moves upwards following the jumper on their ascent so:
 a) They will not fall 10 Feet down as the trampoline is no longer 10 feet below.
 b) To reach 10 Feet above the trampoline requires more force as the jumper has to travel further up to create the 10 foot distance

My understanding is therefore that with a static ground the plane can fly horizontally but if the earth was rushing up towards it it would need to be ascending to maintain it's height above it
Title: Re: Why is the Earth accelerating at 9.8 metres per second?
Post by: Curious Squirrel on September 21, 2017, 02:09:59 PM
Indeed. Does this also mean that when an aeroplane is flying apparently at a stable 30,000 feet it is in actual fact in ascent as surely if it was travelling on a level plain it would be hit by the upwards earth?
[...]
I gathered Junker..........I will have to wait to see if any alternative explanations are offered in this thread as the Wiki or FAQ does not explain enough.
This is explained both in our Wiki (https://wiki.tfes.org/Universal_Acceleration#Equivalence_Principle) and on Wikipedia (https://en.wikipedia.org/wiki/Equivalence_principle). In short: generating enough lift to avoid a flat plane accelerating at 9.8%5Cfrac%7Bm%7D%7Bs%5E2%7D is equivalent and indistinguishable from generating enough lift to prevent an aeroplane from crashing down due to the 9.8%5Cfrac%7Bm%7D%7Bs%5E2%7D acceleration caused by RET's gravitation.

Thanks for responding, I don't follow that they are the same. On RE the ground remains at the same level whilst the plane flies above it, generating thrust to avoid the pull of gravity. On FE the plane is still in the air but crucially the ground is NOT static, it is rushing up towards the plane.

A trampoline jump: RE- a certain force is applied by the jumper to raise themselves 10 feet above the trampoline, then they fall down 10 feet back to trampoline

FE - The trampoline moves upwards following the jumper on their ascent so:
 a) They will not fall 10 Feet down as the trampoline is no longer 10 feet below.
 b) To reach 10 Feet above the trampoline requires more force as the jumper has to travel further up to create the 10 foot distance

My understanding is therefore that with a static ground the plane can fly horizontally but if the earth was rushing up towards it it would need to be ascending to maintain it's height above it
While I agree it's a little weird to really think about, always remember that (roughly) "Acceleration is/can be indistinguishable from a gravitational field" according to part of Relativity. It doesn't help a ton in trying to think about it imo, but it is there. Remember, you and the trampoline we're both going at that acceleration before you jumped. The trampoline added to it (thus resulting in going up) and since you no longer had a force being applied to you, your acceleration decayed until you 'fell' back down onto the trampoline (or the ground) once again and were instantly and forcibly restored to the speed of acceleration of the Earth.
Title: Re: Why is the Earth accelerating at 9.8 metres per second?
Post by: 3DGeek on September 21, 2017, 03:21:28 PM
Thanks for responding, I don't follow that they are the same.
...
My understanding is therefore that with a static ground the plane can fly horizontally but if the earth was rushing up towards it it would need to be ascending to maintain it's height above it

HINT: When both the FE'ers *AND* the RE'ers tell you that you're wrong (and that's what's precisely what's happening here!) you should probably stop talking and go look it up someplace.

Aside from some subtle problems relating to the variability of gravity and things like tidal effects - universal acceleration does "work".  (Although - I still have no clue why the FE'ers bother with it).

If UA was how the world ACTUALLY worked - your daily experience of life (jumping, trampolines, airplanes, balloons, etc) would seem almost exactly the same as it does with gravity and a round planet.

There are differences which disprove UA...but they are FAR too subtle to see without delicate equipment and a lot of Airline Miles to burn!

Sorry you can't grasp it...but don't feel bad about it.   It took Albert Einstein's brain to figure it out for us.
Title: Re: Why is the Earth accelerating at9.8 metres per second?
Post by: devils advocate on September 21, 2017, 08:46:12 PM
Haha thanks for explaining CS and 3D, from what I've read of you two over my FES experience so far I trust that your knowledge of physics beats mine so I'll back down on this one. Thanks for taking the time to explain :-)
Title: Re: Why is the Earth accelerating at9.8 metres per second?
Post by: 3DGeek on September 26, 2017, 05:17:32 PM
Haha thanks for explaining CS and 3D, from what I've read of you two over my FES experience so far I trust that your knowledge of physics beats mine so I'll back down on this one. Thanks for taking the time to explain :-)

There is one distinction you might want to think about though.  Acceleration requires energy to make it happen.  Gravitation does not.   What exactly powers the Earth through this crazy amount of acceleration?

Title: Re: Why is the Earth accelerating at9.8 metres per second?
Post by: juner on September 26, 2017, 06:27:51 PM
Haha thanks for explaining CS and 3D, from what I've read of you two over my FES experience so far I trust that your knowledge of physics beats mine so I'll back down on this one. Thanks for taking the time to explain :-)
Acceleration requires energy to make it happen.  Gravitation does not.

What does gravitation require to make it happen, then?
Title: Re: Why is the Earth accelerating at9.8 metres per second?
Post by: mtnman on September 26, 2017, 09:53:35 PM

What does gravitation require to make it happen, then?
Mass
Title: Re: Why is the Earth accelerating at9.8 metres per second?
Post by: 3DGeek on September 26, 2017, 09:57:55 PM
Acceleration requires energy to make it happen.  Gravitation does not.

What does gravitation require to make it happen, then?

It is a VERY common misapprehension that a FORCE requires ENERGY.   It does not.  When you attach a magnet to your refrigerator, it stays stuck there forever without "running down" because it's not consuming any energy to be there.   When a book sits on a table, the gravitational pull of the book on the Earth doesn't run down - and the equal-and-opposite force of the table pushing up on the book doesn't require energy either.   The book sits on the table without either consuming energy.

Gravity is like magnetism in that regard - both produce a force without energy expenditure.

HOWEVER:  "Work" is "Force-through-a-distance" - and "Work" is "Energy".   So as soon as something moves - energy is required to make it happen.   So if the Earth is physically moving as it accelerates upwards - then for as long as it accelerates, some energy is being expended somewhere.

With gravity - if you LIFT a book to some height, then you're providing energy with your muscles to move the book.  The book retains that energy as "gravitational potential energy" - and if you drop it, that energy will turn into kinetic energy as the book accelerates towards the ground - and then into heat and sound energy as it impacts.

But your Flat Earth is being accelerated continually - accumulating more and more kinetic energy - which DOES require some input of energy from someplace.

Now - you might argue for magic - or some "unobtainium" that carries limitless energy within it...but you DO need some kind of an explanation for where this energy has been coming from for the last god-knows billions of years.
Title: Re: Why is the Earth accelerating at 9.8 metres per second?
Post by: JHelzer on September 26, 2017, 11:03:08 PM
Thanks for responding, I don't follow that they are the same. On RE the ground remains at the same level whilst the plane flies above it, generating thrust to avoid the pull of gravity. On FE the plane is still in the air but crucially the ground is NOT static, it is rushing up towards the plane.

A trampoline jump: RE- a certain force is applied by the jumper to raise themselves 10 feet above the trampoline, then they fall down 10 feet back to trampoline

FE - The trampoline moves upwards following the jumper on their ascent so:
 a) They will not fall 10 Feet down as the trampoline is no longer 10 feet below.
 b) To reach 10 Feet above the trampoline requires more force as the jumper has to travel further up to create the 10 foot distance

My understanding is therefore that with a static ground the plane can fly horizontally but if the earth was rushing up towards it it would need to be ascending to maintain it's height above it
devil's advocate, I'll try to explain how this works as I see it...

Hope this makes sense and is helpful.
Title: Re: Why is the Earth accelerating at9.8 metres per second?
Post by: Bobgreco61 on September 26, 2017, 11:12:28 PM
I have read the Wiki and FAQ; (UA asserts that the Earth is accelerating 'upward' at a constant rate of 9.8m/s^2)

But am struggling to find either a reason why or proof that the earth IS accelerating at 9.8 metres per second:

Wiki states: "The are several explanations for UA. As it is difficult for proponents of Flat Earth Theory to obtain grant money for scientific research, it is nigh on impossible to determine which of these theories is correct."

I was hoping that a FE could explain why this theory exists without this reasoning and evidence? Why does it make sense to you and how did this exact speed come about? As for the several explanations for UA; what are they please as I could not see t
many thanks
Title: Re: Why is the Earth accelerating at9.8 metres per second?
Post by: juner on September 27, 2017, 12:56:40 AM
What does gravitation require to make it happen, then?
Mass
Hmm, I do believe there was a guy who thought there was some equivalence between energy and mass. Interesting.


Acceleration requires energy to make it happen.  Gravitation does not.

What does gravitation require to make it happen, then?

It is a VERY common misapprehension that a FORCE requires ENERGY.   It does not.  When you attach a magnet to your refrigerator, it stays stuck there forever without "running down" because it's not consuming any energy to be there.   When a book sits on a table, the gravitational pull of the book on the Earth doesn't run down - and the equal-and-opposite force of the table pushing up on the book doesn't require energy either.   The book sits on the table without either consuming energy.

Gravity is like magnetism in that regard - both produce a force without energy expenditure.

HOWEVER:  "Work" is "Force-through-a-distance" - and "Work" is "Energy".   So as soon as something moves - energy is required to make it happen.   So if the Earth is physically moving as it accelerates upwards - then for as long as it accelerates, some energy is being expended somewhere.

With gravity - if you LIFT a book to some height, then you're providing energy with your muscles to move the book.  The book retains that energy as "gravitational potential energy" - and if you drop it, that energy will turn into kinetic energy as the book accelerates towards the ground - and then into heat and sound energy as it impacts.

But your Flat Earth is being accelerated continually - accumulating more and more kinetic energy - which DOES require some input of energy from someplace.

Now - you might argue for magic - or some "unobtainium" that carries limitless energy within it...but you DO need some kind of an explanation for where this energy has been coming from for the last god-knows billions of years.

You didn't answer the question. You claim gravitation does not require energy to make it happen. So, what makes gravitation happen?
Title: Re: Why is the Earth accelerating at9.8 metres per second?
Post by: 3DGeek on September 27, 2017, 01:10:10 AM
What does gravitation require to make it happen, then?
Mass
Hmm, I do believe there was a guy who thought there was some equivalence between energy and mass. Interesting.


Acceleration requires energy to make it happen.  Gravitation does not.

What does gravitation require to make it happen, then?

It is a VERY common misapprehension that a FORCE requires ENERGY.   It does not.  When you attach a magnet to your refrigerator, it stays stuck there forever without "running down" because it's not consuming any energy to be there.   When a book sits on a table, the gravitational pull of the book on the Earth doesn't run down - and the equal-and-opposite force of the table pushing up on the book doesn't require energy either.   The book sits on the table without either consuming energy.

Gravity is like magnetism in that regard - both produce a force without energy expenditure.

HOWEVER:  "Work" is "Force-through-a-distance" - and "Work" is "Energy".   So as soon as something moves - energy is required to make it happen.   So if the Earth is physically moving as it accelerates upwards - then for as long as it accelerates, some energy is being expended somewhere.

With gravity - if you LIFT a book to some height, then you're providing energy with your muscles to move the book.  The book retains that energy as "gravitational potential energy" - and if you drop it, that energy will turn into kinetic energy as the book accelerates towards the ground - and then into heat and sound energy as it impacts.

But your Flat Earth is being accelerated continually - accumulating more and more kinetic energy - which DOES require some input of energy from someplace.

Now - you might argue for magic - or some "unobtainium" that carries limitless energy within it...but you DO need some kind of an explanation for where this energy has been coming from for the last god-knows billions of years.

You didn't answer the question. You claim gravitation does not require energy to make it happen. So, what makes gravitation happen?

Ah - that game my 5 year old granddaughter plays...they ask a question, you answer, they say "Why?" so you explain and they say "Why?" so you explain that and then they say "Why?" again.

Ultimately, we know what mass does - it makes gravity.  And that's because of the curvature of space-time and...well, I guess we ran out of answers for "Why?".

But unless I'm very much mistaken, FET has gravity too...it's the only way you can (half-assedly) explain half of the tides...I suppose that's quarter-assedly.  So we don't have to play this game...FET and RET both have gravity - neither of us can get passed "curving space-time".

But NOWHERE do we get free energy - we even have one of the best established laws of physics that says "No Free Energy!!"
Title: Re: Why is the Earth accelerating at9.8 metres per second?
Post by: Horhang on September 27, 2017, 01:29:07 AM
If Earth is accelerating up at 9.8 m/s/s that means in 24 hours Earth would have accelerated by:
60s/min x 60 min/hr x 24hr/day x 9.8 m/s/s = 846,720 m/s/s per day.
Which means within 360 days the earth would have accelerated by 846,720 x 360 = 304,819,200 m/s/s.
Which means each year Earth would have accelerated beyond the speed of light. Relativity tells us this is impossible so what is the solution to this dilemma? Alternatively what is the source of energy driving the continued acceleration of Earth?
Title: Re: Why is the Earth accelerating at9.8 metres per second?
Post by: xenotolerance on September 27, 2017, 01:38:51 AM
One of theoretical physics' top questions is indeed, What causes gravity? Ask Rovelli (https://en.wikipedia.org/wiki/Loop_quantum_gravity):
Quote
According to Einstein, gravity is not a force – it is a property of space-time itself. Loop quantum gravity is an attempt to develop a quantum theory of gravity based directly on Einstein's geometrical formulation.

One way to put it is that gravity does not need energy because it is energy, but the only thing it can do is move things from high altitudes to low altitudes. In these terms, it is also known as potential energy. But I do recommend studying loop quantum gravity, because it rules pretty hard, although it is predicated on general relativity and quantum physics with which flat earth does not play well.

Speaking of which - finite acceleration within one reference frame can never reach the speed of light, something that is brought up early and often by universal accelerator proponents when people raise that objection. However, this limit is famously part of special relativity which, being based on Newton's theory of gravity, obviates the universal accelerator idea completely. Relativity replaces Newton's theory by, in brief, combining certain variables, but it doesn't eschew gravity at all.
Title: Re: Why is the Earth accelerating at9.8 metres per second?
Post by: juner on September 27, 2017, 02:22:20 AM
Ah - that game my 5 year old granddaughter plays...they ask a question, you answer, they say "Why?" so you explain and they say "Why?" so you explain that and then they say "Why?" again.
Funny, there wasn't an explanation for the question I asked at all. I would suggest you go back and read the thread again to clear up your apparent misunderstanding, then maybe try again.

Ultimately, we know what mass does - it makes gravity.  And that's because of the curvature of space-time and...well, I guess we ran out of answers for "Why?".
You could have just said you don't know to begin with. Nothing wrong with that. Although mass and energy tend to have a bit of an equivalence, especially in relativity. Of course, you already know this.

But unless I'm very much mistaken, FET has gravity too...
Nope.

But NOWHERE do we get free energy - we even have one of the best established laws of physics that says "No Free Energy!!"
Correct. Again, it is almost like there is an equivalence somewhere in there.


If Earth is accelerating up at 9.8 m/s/s that means in 24 hours Earth would have accelerated by:
60s/min x 60 min/hr x 24hr/day x 9.8 m/s/s = 846,720 m/s/s per day.
Which means within 360 days the earth would have accelerated by 846,720 x 360 = 304,819,200 m/s/s.
Which means each year Earth would have accelerated beyond the speed of light. Relativity tells us this is impossible so what is the solution to this dilemma? Alternatively what is the source of energy driving the continued acceleration of Earth?
Special relativity is your solution. I would suggest looking into it and you will find that you can undergo constant acceleration and asymptotically approach the speed of light, but never reach it.

Title: Re: Why is the Earth accelerating at9.8 metres per second?
Post by: 3DGeek on September 27, 2017, 03:05:41 AM
If Earth is accelerating up at 9.8 m/s/s that means in 24 hours Earth would have accelerated by:
60s/min x 60 min/hr x 24hr/day x 9.8 m/s/s = 846,720 m/s/s per day.
Which means within 360 days the earth would have accelerated by 846,720 x 360 = 304,819,200 m/s/s.
Which means each year Earth would have accelerated beyond the speed of light. Relativity tells us this is impossible so what is the solution to this dilemma? Alternatively what is the source of energy driving the continued acceleration of Earth?

Argh...STOP SAYING THAT!  You clearly don't understand relativity.

That is NOT the problem here.   Relativity says that from the perspective of people here on Earth, we can indeed keep accelerating indefinitely.   From the perspective of some hypothetical "stationary" being off to the side, watching this happen, they'd see the clocks of people here slowing down more and more (because of time dilation) and the length of the earth (and our rulers) stretching out further and further in the direction the Earth is travelling - so that the Earth would indeed never exceed light speed.

There are plenty of OTHER problems with FET - but modern physics says that this isn't one of them!
Title: Re: Why is the Earth accelerating at9.8 metres per second?
Post by: Curious Squirrel on September 27, 2017, 03:40:56 AM
But unless I'm very much mistaken, FET has gravity too...
Nope.
Until you actually explain the difference between gravity and gravitation, yes, yes you do. So far you have not other than to simply claim the last thread it came up in already explained it.

I would also note the infinite Earth plane does feature gravity, although it's not the preferred model on THIS site, it is on the other one. Just a note for others.
Title: Re: Why is the Earth accelerating at9.8 metres per second?
Post by: Ga_x2 on September 27, 2017, 06:00:31 AM
But unless I'm very much mistaken, FET has gravity too...
Nope.
I understand you're the ultimate master in hair splitting, but it really makes no sense in this context. Gravity / gravitation, tomato / tomahto, it remains an unexplained mess, that you should clear up beyond a one-liner in the wiki.
I also keep bringing up the Cavendish experiment, in this regard, but I only get crickets in response  (beside a random link from Tom that amused me for days). I understand that bumping threads is not kosher, so if anyone has an answer, it'd be super nice if they'd fish the question a few pages back and have at it.
Besos
Title: Re: Why is the Earth accelerating at9.8 metres per second?
Post by: StinkyOne on September 27, 2017, 12:38:48 PM
If Earth is accelerating up at 9.8 m/s/s that means in 24 hours Earth would have accelerated by:
60s/min x 60 min/hr x 24hr/day x 9.8 m/s/s = 846,720 m/s/s per day.
Which means within 360 days the earth would have accelerated by 846,720 x 360 = 304,819,200 m/s/s.
Which means each year Earth would have accelerated beyond the speed of light. Relativity tells us this is impossible so what is the solution to this dilemma? Alternatively what is the source of energy driving the continued acceleration of Earth?

I'm a REer, but the FEers will bring up the Lorentz factor which basically says that an object with mass can never be accelerated to the speed of light. What I don't think I've seen an explanation for is the fact that their energy source would need to increase exponentially to keep a steady acceleration. The reason you can't get a massive object to the speed of light is because it would require an infinite amount of energy. Once again, just another flat Earth hack using only half of the facts.
Title: Re: Why is the Earth accelerating at9.8 metres per second?
Post by: juner on September 27, 2017, 02:31:59 PM
I'm a REer, but the FEers will bring up the Lorentz factor which basically says that an object with mass can never be accelerated to the speed of light. What I don't think I've seen an explanation for is the fact that their energy source would need to increase exponentially to keep a steady acceleration. The reason you can't get a massive object to the speed of light is because it would require an infinite amount of energy. Once again, just another flat Earth hack using only half of the facts.

It seems you are confusing proper acceleration with coordinate acceleration. UA may have flaws, but this isn't one of them.
Title: Re: Why is the Earth accelerating at9.8 metres per second?
Post by: devils advocate on September 27, 2017, 02:50:58 PM

It seems you are confusing proper acceleration with coordinate acceleration. UA may have flaws, but this isn't one of them.

Please then Junker can you shed some light on why the earth is accelerating at all? The start of this thread was asking for an explanation into the theory behind UA but so far there has been nothing offered as to what indicates that this is occurring, what the evidence is for it and what force started it. Also is everything in the universe moving in the same direction also.
Title: Re: Why is the Earth accelerating at9.8 metres per second?
Post by: Curious Squirrel on September 27, 2017, 03:29:33 PM

It seems you are confusing proper acceleration with coordinate acceleration. UA may have flaws, but this isn't one of them.

Please then Junker can you shed some light on why the earth is accelerating at all? The start of this thread was asking for an explanation into the theory behind UA but so far there has been nothing offered as to what indicates that this is occurring, what the evidence is for it and what force started it. Also is everything in the universe moving in the same direction also.
Basically, the UA exists because FE needs a way to explain RE's gravity. That's it. They have no experimental evidence for it, it's predicated on the idea that you need a way to have gravity. Thus, you get the UA (or the infinite plane Earth) to supply that to the flat Earth https://wiki.tfes.org/Universal_Acceleration Note they claim it's being done by Dark Energy, which just isn't well understood and mere conjecture on the part of all involved (with Dark Energy I mean, it's pretty specious even in the RE realm still as I recall, essentially an idea to explain observed oddities that is being worked on.)
Title: Re: Why is the Earth accelerating at9.8 metres per second?
Post by: juner on September 27, 2017, 03:36:01 PM
They have no experimental evidence for it, it's predicated on the idea that you need a way to have gravity.

I would suggest holding something (preferable not easily broken) in your hand a meter or more above the ground and letting it go. You can observe acceleration occur right before your eyes.
Title: Re: Why is the Earth accelerating at9.8 metres per second?
Post by: Curious Squirrel on September 27, 2017, 03:43:07 PM
They have no experimental evidence for it, it's predicated on the idea that you need a way to have gravity.

I would suggest holding something (preferable not easily broken) in your hand a meter or more above the ground and letting it go. You can observe acceleration occur right before your eyes.
Which is indistinguishable from gravity. So as not to upset Junker's sensitive little pedantic meter let me be clear. They have no direct evidence that shows something is accelerating the Earth, rather than an effect happening called gravity. The wiki page literally calls out that the two effects would be functionally the same via Relativity. But there's nothing (that I've ever seen shown) that would prove it's acceleration, and not gravity. As a reminder, the other hypothesis for FE asserts an infinite plane with normal gravity, as also listed on their wiki.
Title: Re: Why is the Earth accelerating at9.8 metres per second?
Post by: juner on September 27, 2017, 04:04:49 PM
Which is indistinguishable from gravity. So as not to upset Junker's sensitive little pedantic meter let me be clear. They have no direct evidence that shows something is accelerating the Earth, rather than an effect happening called gravity.

I think acceleration is much more easily-observed than gravity. Anyone can do it in a variety of ways. Observing gravity, though... That is another story.
Title: Re: Why is the Earth accelerating at9.8 metres per second?
Post by: Ga_x2 on September 27, 2017, 04:16:54 PM
Which is indistinguishable from gravity. So as not to upset Junker's sensitive little pedantic meter let me be clear. They have no direct evidence that shows something is accelerating the Earth, rather than an effect happening called gravity.

I think acceleration is much more easily-observed than gravity. Anyone can do it in a variety of ways. Observing gravity, though... That is another story.
caaaaaveeeendiiiiish!  8)
Title: Re: Why is the Earth accelerating at9.8 metres per second?
Post by: Curious Squirrel on September 27, 2017, 04:22:12 PM
Which is indistinguishable from gravity. So as not to upset Junker's sensitive little pedantic meter let me be clear. They have no direct evidence that shows something is accelerating the Earth, rather than an effect happening called gravity.

I think acceleration is much more easily-observed than gravity. Anyone can do it in a variety of ways. Observing gravity, though... That is another story.
Yes, but saying "Watch this thing go from your hand to the ground. Acceleration!" is not one of them. Again, no evidence of an accelerating Earth as opposed to a globe Earth with gravity. UA seems designed with that literally in mind. It's not about observing acceleration vs observing gravity, it's about observing the Earth accelerating, vs seeing a round Earth that exhibits the effect we know as gravity. From a frame of reference upon the Earth, there is no difference. For being so pedantic you sure have a tough time grasping linguistic differences sometimes.
Title: Re: Why is the Earth accelerating at9.8 metres per second?
Post by: juner on September 27, 2017, 04:49:51 PM
caaaaaveeeendiiiiish!  8)
You think that is more easily observed than acceleration? I would suggest that you maybe do some research before posting. It will help to keep you from looking as ignorant as you do here.


Yes, but saying "Watch this thing go from your hand to the ground. Acceleration!" is not one of them.
Except it literally is an example of acceleration. Unless you are actually trying to argue that it isn't, which if you are then I suppose then isn't much left to discuss.


Again, no evidence of an accelerating Earth as opposed to a globe Earth with gravity.
Again, no evidence of a globe Earth with gravity as opposed to an accelerating Earth. See, I can do it too...


UA seems designed with that literally in mind. It's not about observing acceleration vs observing gravity, it's about observing the Earth accelerating, vs seeing a round Earth that exhibits the effect we know as gravity.
Cool, so how can we go about observing gravity? What is the mechanism that makes it function?


For being so pedantic you sure have a tough time grasping linguistic differences sometimes.
I am sorry if you are having trouble understanding simple concepts. I can try be even more simple with the explanations if you would prefer.
Title: Re: Why is the Earth accelerating at9.8 metres per second?
Post by: Curious Squirrel on September 27, 2017, 05:49:19 PM
Again, no evidence of an accelerating Earth as opposed to a globe Earth with gravity.
Again, no evidence of a globe Earth with gravity as opposed to an accelerating Earth. See, I can do it too...

UA seems designed with that literally in mind. It's not about observing acceleration vs observing gravity, it's about observing the Earth accelerating, vs seeing a round Earth that exhibits the effect we know as gravity.
Cool, so how can we go about observing gravity? What is the mechanism that makes it function?

The bolded is literally what I'm saying. The observation of a ball going from your hand to the ground isn't evidence either way based on how UA has been put forward. You presented it as though it was evidence of UA. It isn't a conclusive experiment in either direction.

The ease or not of replicating the Cavendish experiment has no bearing on UA not having any direct evidence for it. Can you show us the force that is making the Earth accelerate? Can you conclusively show us that it's the Earth accelerating towards the ball when we let got of it and not the ball accelerating towards the Earth? If you can I would love to see it.
Title: Re: Why is the Earth accelerating at9.8 metres per second?
Post by: Ga_x2 on September 27, 2017, 06:07:33 PM
caaaaaveeeendiiiiish!  8)
You think that is more easily observed than acceleration? I would suggest that you maybe do some research before posting. It will help to keep you from looking as ignorant as you do here.
lol ok. Please explain the Cavendish experiment within the constraint of UA.
Title: Re: Why is the Earth accelerating at9.8 metres per second?
Post by: juner on September 27, 2017, 06:35:39 PM
Can you show us the force that is making the Earth accelerate? Can you conclusively show us that it's the Earth accelerating towards the ball when we let got of it and not the ball accelerating towards the Earth? If you can I would love to see it.
Not anymore than RE can show what makes gravity function, I suppose. But if you have some evidence to the contrary, I'd love to see it.


lol ok. Please explain the Cavendish experiment within the constraint of UA.
Nice deflection of the question. I would suggest you review the FAQ and the wiki.
Title: Re: Why is the Earth accelerating at9.8 metres per second?
Post by: Curious Squirrel on September 27, 2017, 06:57:46 PM
Can you show us the force that is making the Earth accelerate? Can you conclusively show us that it's the Earth accelerating towards the ball when we let got of it and not the ball accelerating towards the Earth? If you can I would love to see it.
Not anymore than RE can show what makes gravity function, I suppose. But if you have some evidence to the contrary, I'd love to see it.
Then stop claiming dropping a ball is evidence for UA. It's evidence for something bringing the ball from your hand to the ground.
Title: Re: Why is the Earth accelerating at9.8 metres per second?
Post by: juner on September 27, 2017, 07:00:36 PM
Can you show us the force that is making the Earth accelerate? Can you conclusively show us that it's the Earth accelerating towards the ball when we let got of it and not the ball accelerating towards the Earth? If you can I would love to see it.
Not anymore than RE can show what makes gravity function, I suppose. But if you have some evidence to the contrary, I'd love to see it.
Then stop claiming dropping a ball is evidence for UA. It's evidence for something bringing the ball from your hand to the ground.

Did I claim it was evidence for UA? I simply said it was evidence of acceleration.
Title: Re: Why is the Earth accelerating at9.8 metres per second?
Post by: mtnman on September 27, 2017, 07:04:13 PM
Can you show us the force that is making the Earth accelerate? Can you conclusively show us that it's the Earth accelerating towards the ball when we let got of it and not the ball accelerating towards the Earth? If you can I would love to see it.
Not anymore than RE can show what makes gravity function, I suppose. But if you have some evidence to the contrary, I'd love to see it.

Newton explained the effects of gravity in the 1600's and Einstein explained how it worked in the early 1900's. Over the last hundred years, more and more pieces of Einstein's theories have been confirmed in various experimental tests. Here is a summary. https://www.space.com/17661-theory-general-relativity.html (https://www.space.com/17661-theory-general-relativity.html)

One caveat, I have not seen it explicitly mentioned anywhere, but I am assuming Einstein was a RE believer.

Title: Re: Why is the Earth accelerating at9.8 metres per second?
Post by: Curious Squirrel on September 27, 2017, 07:12:12 PM
Can you show us the force that is making the Earth accelerate? Can you conclusively show us that it's the Earth accelerating towards the ball when we let got of it and not the ball accelerating towards the Earth? If you can I would love to see it.
Not anymore than RE can show what makes gravity function, I suppose. But if you have some evidence to the contrary, I'd love to see it.
Then stop claiming dropping a ball is evidence for UA. It's evidence for something bringing the ball from your hand to the ground.

Did I claim it was evidence for UA? I simply said it was evidence of acceleration.

No, you didn't. Context. Learn to understand how it adjusts what you say, and to be clear in what you say when it's not in the context of what's going on if that's truly what you meant.
Title: Re: Why is the Earth accelerating at9.8 metres per second?
Post by: 3DGeek on September 27, 2017, 07:16:37 PM
Again, no evidence of an accelerating Earth as opposed to a globe Earth with gravity.
Again, no evidence of a globe Earth with gravity as opposed to an accelerating Earth. See, I can do it too...

Ah - but there you're wrong.

How does UA explain that objects fall to the ground faster at the poles than at the equator?

In RET, it's a natural consequence of the rotation of the earth producing a centrifugal force that slightly opposes gravity at the equator, but not at the poles.

UA can't do that because the entire FE has to accelerate at the same exact rate or it'll tear itself apart.

While you're about it - tell us how there can be less "acceleration" at the tops of mountains....oh - and more where are dense rock formations underground.

(I still don't understand why FE'ers here don't just go with "gravity on an infinite plane" - it makes life so much easier.)
Title: Re: Why is the Earth accelerating at9.8 metres per second?
Post by: juner on September 27, 2017, 07:26:51 PM
No, you didn't. Context. Learn to understand how it adjusts what you say, and to be clear in what you say when it's not in the context of what's going on if that's truly what you meant.
Actually, I did. Unless you can show me where I said it was evidence of UA, please stop making things up. Thanks.


Ah - but there you're wrong.

How does UA explain that objects fall to the ground faster at the poles than at the equator?

In RET, it's a natural consequence of the rotation of the earth producing a centrifugal force that slightly opposes gravity at the equator, but not at the poles.

UA can't do that because the entire FE has to accelerate at the same exact rate or it'll tear itself apart.

While you're about it - tell us how there can be less "acceleration" at the tops of mountains....oh - and more where are dense rock formations underground.

(I still don't understand why FE'ers here don't just go with "gravity on an infinite plane" - it makes life so much easier.)

Already addressed in the wiki/FAQ, friend. It is the same answer, even if you don't like it.
Title: Re: Why is the Earth accelerating at9.8 metres per second?
Post by: Curious Squirrel on September 27, 2017, 07:46:01 PM
No, you didn't. Context. Learn to understand how it adjusts what you say, and to be clear in what you say when it's not in the context of what's going on if that's truly what you meant.
Actually, I did. Unless you can show me where I said it was evidence of UA, please stop making things up. Thanks.
I said
Quote
Basically, the UA exists because FE needs a way to explain RE's gravity. That's it. They have no experimental evidence for it, it's predicated on the idea that you need a way to have gravity. Thus, you get the UA (or the infinite plane Earth) to supply that to the flat Earth https://wiki.tfes.org/Universal_Acceleration Note they claim it's being done by Dark Energy, which just isn't well understood and mere conjecture on the part of all involved (with Dark Energy I mean, it's pretty specious even in the RE realm still as I recall, essentially an idea to explain observed oddities that is being worked on.)

To which you replied
Quote
Quote
They have no experimental evidence for it, it's predicated on the idea that you need a way to have gravity.
I would suggest holding something (preferable not easily broken) in your hand a meter or more above the ground and letting it go. You can observe acceleration occur right before your eyes.

Context suggests you are offering that observation as evidence for UA. Whether you understand that or not appears to be a different question. Now obviously you intended "You can observe acceleration occur right before your eyes" to mean you were talking strictly about acceleration, or more precisely I think, this allows you to twist the meaning any way you want in the future. But if you weren't commenting about the UA discussion, why bother saying anything? You certainly have no problems staying silent in other places. Therefore context cues suggest you were offering experimental evidence of the acceleration of the Earth.
Title: Re: Why is the Earth accelerating at9.8 metres per second?
Post by: juner on September 27, 2017, 07:48:58 PM
To which you replied
Quote
I would suggest holding something (preferable not easily broken) in your hand a meter or more above the ground and letting it go. You can observe acceleration occur right before your eyes.

So no claim beyond observing acceleration. Gotcha, thanks for proving my point and showing where you were incorrect. You are welcome to interpret things how you like, but that certainly doesn't make them correct. Maybe we can get back to the topic now that you have located your error.

Title: Re: Why is the Earth accelerating at9.8 metres per second?
Post by: Curious Squirrel on September 27, 2017, 08:00:28 PM
To which you replied
Quote
I would suggest holding something (preferable not easily broken) in your hand a meter or more above the ground and letting it go. You can observe acceleration occur right before your eyes.

So no claim beyond observing acceleration. Gotcha, thanks for proving my point and showing where you were incorrect. You are welcome to interpret things how you like, but that certainly doesn't make them correct. Maybe we can get back to the topic now that you have located your error.
Bravo junker, truly you are not just a master at taking things out of context to fit your narrative, but obviously a top notch troll as well. Bravo, yes let's see about getting back on track, where were we...
Title: Re: Why is the Earth accelerating at9.8 metres per second?
Post by: 3DGeek on September 27, 2017, 08:17:08 PM
No, you didn't. Context. Learn to understand how it adjusts what you say, and to be clear in what you say when it's not in the context of what's going on if that's truly what you meant.
Actually, I did. Unless you can show me where I said it was evidence of UA, please stop making things up. Thanks.


Ah - but there you're wrong.

How does UA explain that objects fall to the ground faster at the poles than at the equator?

In RET, it's a natural consequence of the rotation of the earth producing a centrifugal force that slightly opposes gravity at the equator, but not at the poles.

UA can't do that because the entire FE has to accelerate at the same exact rate or it'll tear itself apart.

While you're about it - tell us how there can be less "acceleration" at the tops of mountains....oh - and more where are dense rock formations underground.

(I still don't understand why FE'ers here don't just go with "gravity on an infinite plane" - it makes life so much easier.)

Already addressed in the wiki/FAQ, friend. It is the same answer, even if you don't like it.

As usual, Junker gets to the limits of his ability to explain then punts to the Wiki...which (as usual) contains nothing whatever of value.

Well,  certainly doesn't explain it - the word "equator" isn't mentioned even once.

The appalling lack of science on this page is astounding! "This force is known as "Universal Acceleration"" - why would you name a force "acceleration"?!  Well - whatever.

The explanation of how this is like RET gravity is also pretty hilarious: "its effect on smaller bodies is negligible (similar to gravity in RET cosmology, which only has a noticeable affect on very large objects)."  In RET, gravity is a force between two objects.  A pebble pulls on the Earth with the exact same force that the Earth pulls on the pebble...Newton's Third Law guys.  The reason the pebble accelerates faster than the Earth is because it's so much lighter and F=ma, so: a=F/m.

Anyway - we don't need to discuss FE'ers inability to understand RET...it's funny when they try - but unimportant here.

There is a tiny bit in there about the reduction in gravity with altitude:

"The gravitational pull of the stars, for example, causes observable tidal effects on Earth."
Er...no!  Why do the tides vary in timing from day to day when the stars are always in the same exact position at the same time every day?  Where to the stars go to when there is a low tide at midnight?  Tides are VERY clearly a lunar phenomenon.

Q: Why does gravity vary with altitude?  A: The moon and stars have a slight gravitational pull.

Well - not really.  This "gravitation" (but not "gravity" - even though we just called it that by mistake!) can't produce both a variable attraction for the tides and a constant attraction for reduced gravity on mountain tops.  One or the other guys.

Then there is another page [url=https://wiki.tfes.org/Celestial_Gravitation]https://wiki.tfes.org/Universal_Acceleration]https://wiki.tfes.org/Celestial_Gravitation]https://wiki.tfes.org/Universal_Acceleration (https://wiki.tfes.org/Universal_Acceleration[/url) certainly doesn't explain it - the word "equator" isn't mentioned even once.

The appalling lack of science on this page is astounding! "This force is known as "Universal Acceleration"" - why would you name a force "acceleration"?!  Well - whatever.

The explanation of how this is like RET gravity is also pretty hilarious: "its effect on smaller bodies is negligible (similar to gravity in RET cosmology, which only has a noticeable affect on very large objects)."  In RET, gravity is a force between two objects.  A pebble pulls on the Earth with the exact same force that the Earth pulls on the pebble...Newton's Third Law guys.  The reason the pebble accelerates faster than the Earth is because it's so much lighter and F=ma, so: a=F/m.

Anyway - we don't need to discuss FE'ers inability to understand RET...it's funny when they try - but unimportant here.

There is a tiny bit in there about the reduction in gravity with altitude:

"The gravitational pull of the stars, for example, causes observable tidal effects on Earth."
Er...no!  Why do the tides vary in timing from day to day when the stars are always in the same exact position at the same time every day?  Where to the stars go to when there is a low tide at midnight?  Tides are VERY clearly a lunar phenomenon.

Q: Why does gravity vary with altitude?  A: The moon and stars have a slight gravitational pull.

Well - not really.  This "gravitation" (but not "gravity" - even though we just called it that by mistake!) can't produce both a variable attraction for the tides and a constant attraction for reduced gravity on mountain tops.  One or the other guys.

Then there is another page https://wiki.tfes.org/Celestial_Gravitation (https://wiki.tfes.org/Celestial_Gravitation) that says "Celestial Gravitation accounts for tides and other gravimetric anomalies across the Earth's plane."...ah..."it accounts for it"...well that's a very complete and comprehensive explanation....**NOT**

It's like - "We've invented a name - and decided that it works - and now we're done!"...very zetetic.

So the Wiki DOES NOT explain a darned thing...although I know Junker wishes it were otherwise.
Title: Re: Why is the Earth accelerating at9.8 metres per second?
Post by: juner on September 27, 2017, 08:27:59 PM
No, you didn't. Context. Learn to understand how it adjusts what you say, and to be clear in what you say when it's not in the context of what's going on if that's truly what you meant.
Actually, I did. Unless you can show me where I said it was evidence of UA, please stop making things up. Thanks.


Ah - but there you're wrong.

How does UA explain that objects fall to the ground faster at the poles than at the equator?

In RET, it's a natural consequence of the rotation of the earth producing a centrifugal force that slightly opposes gravity at the equator, but not at the poles.

UA can't do that because the entire FE has to accelerate at the same exact rate or it'll tear itself apart.

While you're about it - tell us how there can be less "acceleration" at the tops of mountains....oh - and more where are dense rock formations underground.

(I still don't understand why FE'ers here don't just go with "gravity on an infinite plane" - it makes life so much easier.)

Already addressed in the wiki/FAQ, friend. It is the same answer, even if you don't like it.

As usual, Junker gets to the limits of his ability to explain then punts to the Wiki...which (as usual) contains nothing whatever of value.

Well,  certainly doesn't explain it - the word "equator" isn't mentioned even once.

The appalling lack of science on this page is astounding! "This force is known as "Universal Acceleration"" - why would you name a force "acceleration"?!  Well - whatever.

The explanation of how this is like RET gravity is also pretty hilarious: "its effect on smaller bodies is negligible (similar to gravity in RET cosmology, which only has a noticeable affect on very large objects)."  In RET, gravity is a force between two objects.  A pebble pulls on the Earth with the exact same force that the Earth pulls on the pebble...Newton's Third Law guys.  The reason the pebble accelerates faster than the Earth is because it's so much lighter and F=ma, so: a=F/m.

Anyway - we don't need to discuss FE'ers inability to understand RET...it's funny when they try - but unimportant here.

There is a tiny bit in there about the reduction in gravity with altitude:

"The gravitational pull of the stars, for example, causes observable tidal effects on Earth."
Er...no!  Why do the tides vary in timing from day to day when the stars are always in the same exact position at the same time every day?  Where to the stars go to when there is a low tide at midnight?  Tides are VERY clearly a lunar phenomenon.

Q: Why does gravity vary with altitude?  A: The moon and stars have a slight gravitational pull.

Well - not really.  This "gravitation" (but not "gravity" - even though we just called it that by mistake!) can't produce both a variable attraction for the tides and a constant attraction for reduced gravity on mountain tops.  One or the other guys.

Then there is another page [url=https://wiki.tfes.org/Celestial_Gravitation]https://wiki.tfes.org/Universal_Acceleration[\url] certainly doesn't explain it - the word "equator" isn't mentioned even once.

The appalling lack of science on this page is astounding! "This force is known as "Universal Acceleration"" - why would you name a force "acceleration"?!  Well - whatever.

The explanation of how this is like RET gravity is also pretty hilarious: "its effect on smaller bodies is negligible (similar to gravity in RET cosmology, which only has a noticeable affect on very large objects)."  In RET, gravity is a force between two objects.  A pebble pulls on the Earth with the exact same force that the Earth pulls on the pebble...Newton's Third Law guys.  The reason the pebble accelerates faster than the Earth is because it's so much lighter and F=ma, so: a=F/m.

Anyway - we don't need to discuss FE'ers inability to understand RET...it's funny when they try - but unimportant here.

There is a tiny bit in there about the reduction in gravity with altitude:

"The gravitational pull of the stars, for example, causes observable tidal effects on Earth."
Er...no!  Why do the tides vary in timing from day to day when the stars are always in the same exact position at the same time every day?  Where to the stars go to when there is a low tide at midnight?  Tides are VERY clearly a lunar phenomenon.

Q: Why does gravity vary with altitude?  A: The moon and stars have a slight gravitational pull.

Well - not really.  This "gravitation" (but not "gravity" - even though we just called it that by mistake!) can't produce both a variable attraction for the tides and a constant attraction for reduced gravity on mountain tops.  One or the other guys.

Then there is another page [url]https://wiki.tfes.org/Celestial_Gravitation]https://wiki.tfes.org/Celestial_Gravitation]https://wiki.tfes.org/Universal_Acceleration[\url] certainly doesn't explain it - the word "equator" isn't mentioned even once.

The appalling lack of science on this page is astounding! "This force is known as "Universal Acceleration"" - why would you name a force "acceleration"?!  Well - whatever.

The explanation of how this is like RET gravity is also pretty hilarious: "its effect on smaller bodies is negligible (similar to gravity in RET cosmology, which only has a noticeable affect on very large objects)."  In RET, gravity is a force between two objects.  A pebble pulls on the Earth with the exact same force that the Earth pulls on the pebble...Newton's Third Law guys.  The reason the pebble accelerates faster than the Earth is because it's so much lighter and F=ma, so: a=F/m.

Anyway - we don't need to discuss FE'ers inability to understand RET...it's funny when they try - but unimportant here.

There is a tiny bit in there about the reduction in gravity with altitude:

"The gravitational pull of the stars, for example, causes observable tidal effects on Earth."
Er...no!  Why do the tides vary in timing from day to day when the stars are always in the same exact position at the same time every day?  Where to the stars go to when there is a low tide at midnight?  Tides are VERY clearly a lunar phenomenon.

Q: Why does gravity vary with altitude?  A: The moon and stars have a slight gravitational pull.

Well - not really.  This "gravitation" (but not "gravity" - even though we just called it that by mistake!) can't produce both a variable attraction for the tides and a constant attraction for reduced gravity on mountain tops.  One or the other guys.

Then there is another page [url]https://wiki.tfes.org/Celestial_Gravitation (https://wiki.tfes.org/Universal_Acceleration[\url) that says "Celestial Gravitation accounts for tides and other gravimetric anomalies across the Earth's plane."...ah..."it accounts for it"...well that's a very complete and comprehensive explanation....**NOT**

It's like - "We've invented a name - and decided that it works - and now we're done!"...very zetetic.

So the Wiki DOES NOT explain a darned thing...although I know Junker wishes it were otherwise.

Maybe work on your post formatting. It makes this particular wall of text more of an eyesore than most of them. Speaking of your wall of text, I am sorry if you don't like the answer, but it is the same thing I mentioned in my aforementioned post. I get that you just don't like it. Unfortunately, that doesn't really matter.

Also, this is well beyond a Q&A post now, so I will move it to FED.
Title: Re: Why is the Earth accelerating at9.8 metres per second?
Post by: devils advocate on September 27, 2017, 08:47:54 PM
So basically Junker, FE has NO comment to answer the original question. The Wiki dies NOT answer it and your comments are pedantic critiques of our phrases or hair splitting over terms. The earth is round, gravity exists as a result of the spin and there is no evidence or reason to accept the crazy notion that the earth s a snow globe hurtling through your make it up as you go asking universe.....that said, good nitpicking, if you're not already then a career in politics awaits you!
Title: Re: Why is the Earth accelerating at9.8 metres per second?
Post by: juner on September 27, 2017, 08:52:17 PM
So basically Junker, FE has NO comment to answer the original question. The Wiki dies NOT answer it and your comments are pedantic critiques of our phrases or hair splitting over terms.
I'd suggest going back and reading the thread again to clear up your apparent misunderstanding.


...gravity exists as a result of the spin
??


...and there is no evidence or reason to accept the crazy notion that the earth s a snow globe hurtling through your make it up as you go asking universe.....that said, good nitpicking, if you're not already then a career in politics awaits you!
I legitimately have no idea what you are talking about here.
Title: Re: Why is the Earth accelerating at9.8 metres per second?
Post by: Ga_x2 on September 27, 2017, 10:20:25 PM
lol ok. Please explain the Cavendish experiment within the constraint of UA.
Nice deflection of the question. I would suggest you review the FAQ and the wiki.
what would I be deflecting? what is causing Cavendish balls to accelerate toward each other? Not the acceleration of the plane. Might it be.. I don't know... gravity?
Title: Re: Why is the Earth accelerating at9.8 metres per second?
Post by: juner on September 27, 2017, 10:38:49 PM
what would I be deflecting?

Answering the question. Feel free to go and read it again. If you find yourself struggling to find it, just ask and I will do my best to help you.
Title: Re: Why is the Earth accelerating at9.8 metres per second?
Post by: Ga_x2 on September 27, 2017, 10:41:07 PM
what would I be deflecting?

Answering the question. Feel free to go and read it again. If you find yourself struggling to find it, just ask and I will do my best to help you.
Please do. Pretend I'm a dumb kid, you won't miss by much.
Title: Re: Why is the Earth accelerating at9.8 metres per second?
Post by: juner on September 28, 2017, 01:58:40 AM
what would I be deflecting?

Answering the question. Feel free to go and read it again. If you find yourself struggling to find it, just ask and I will do my best to help you.
Please do. Pretend I'm a dumb kid, you won't miss by much.
While this is a low-effort troll job, I will play along for a moment. Please see the quote string:

caaaaaveeeendiiiiish!  8)
You think that is more easily observed than acceleration? I would suggest that you maybe do some research before posting. It will help to keep you from looking as ignorant as you do here.
lol ok. Please explain the Cavendish experiment within the constraint of UA.

As is very easily observed by anyone who can read, you will notice that your low-content post of "caaaaaveeeendiiiiish!  8)" was followed by my simple question of "You think that is more easily observed than acceleration?" Instead of replying to the question, you said "lol ok. Please explain the Cavendish experiment within the constraint of UA." which you will notice doesn't answer the question I asked you at all. You merely deflected with some unrelated request about how the Cavendish is explained within UA (again, please see the quote above if you find yourself still struggling). It is fine if you don't want to answer a question, but playing dumb is a bit intellectually dishonest. It is clear you don't want to be taken seriously, so this generous offering of help was a one time only kind of deal. I do wish you the best in overcoming whatever issue prevented you from comprehending a simple question, and hope that next time you post that you give a bit more effort. Take care, friend!
Title: Re: Why is the Earth accelerating at9.8 metres per second?
Post by: Ga_x2 on September 28, 2017, 05:07:57 AM
Junker, thanks for the effort, but it's still unclear. I hope you are not an elementary school teacher.
What is not easy to observe? You were talking about letting things drop, and stating, correctly, that the acceleration toward The floor is all you can really observe, good luck proving it's gravity. Now, the Cavendish experiment detects an attraction between masses. It's rather beside the point how accurately it does, if that's what you mean. You need to explain what is causing that attraction. That's all there is to my "low content comment". At least it got your attention, when I pose the question seriously I get crickets...
Title: Re: Why is the Earth accelerating at9.8 metres per second?
Post by: 3DGeek on September 28, 2017, 01:52:08 PM
Feel free to go and read it again. If you find yourself struggling to find it, just ask and I will do my best to help you.

Oh!  Cool!  So I've read the Wiki and I'm definitely struggling with it - so I'm asking you to do your best to help me.  Evidently I just needed to do the "Jeopardy!" thing where I phrase my answers in the form of a question!

So I just read this page: https://wiki.tfes.org/Universal_Acceleration and I'm struggling with it:

1) It says: "This force is known as "Universal Acceleration"" - how can a force be an acceleration?   Force is defined as acceleration times mass.  Which mass is involved here?  The FE is claimed to be infinite in extent - so the mass must be infinite - doesn't this mean that the acceleration must be zero?!?

2) It says: "its effect on smaller bodies is negligible (similar to gravity in RET cosmology, which only has a noticeable affect on very large objects)."  But in RET, gravity is a force between two objects.  A pebble pulls on the Earth with the exact same force that the Earth pulls on the pebble...Newton's Third Law guys.  The reason the pebble accelerates faster than the Earth is because it's so much lighter and F=ma, so: a=F/m.  So what's the rule in FET?

3) "The gravitational pull of the stars, for example, causes observable tidal effects on Earth."  So could you please explain to me why the tides vary in timing from day to day when the stars are always in the same exact position at any given time of day?  Where do the stars go to when there is a low tide at midnight?  I can still see them.  Did their gravity gravitation turn off for some reason?  Why are there two high and two low tides each day.  This is not explained anywhere on the Wiki.

4) "Q: Why does gravity vary with altitude?  A: The moon and stars have a slight gravitational pull."  How doe this "gravitation" (but not "gravity" - even though you just called it that by mistake!) produce both a variable attraction for the tides and a constant attraction for reduced gravity on mountain tops?

Then there is another page https://wiki.tfes.org/Celestial_Gravitation ...

5) It says "Celestial Gravitation accounts for tides and other gravimetric anomalies across the Earth's plane."   How exactly does it "account for it"?  This isn't explained anywhere I could find.

Thanks in advance for your help!  (You know, you should really stick to saying "Just read the Wiki" and "False" and let Tom take the hits for the gaping holes in FET.)

Title: Re: Why is the Earth accelerating at9.8 metres per second?
Post by: juner on September 28, 2017, 02:29:51 PM
I hope you are not an elementary school teacher.
I am not, but I sure feel like one after that exchange. Although, I imagine elementary students wouldn't struggle as much as you have here. I suppose I would be a remedial elementary teacher in this case.


Feel free to go and read it again. If you find yourself struggling to find it, just ask and I will do my best to help you.

Oh!  Cool!  So I've read the Wiki and I'm definitely struggling with it - so I'm asking you to do your best to help me.  Evidently I just needed to do the "Jeopardy!" thing where I phrase my answers in the form of a question!

So I just read this page: https://wiki.tfes.org/Universal_Acceleration and I'm struggling with it:

1) It says: "This force is known as "Universal Acceleration"" - how can a force be an acceleration?   Force is defined as acceleration times mass.  Which mass is involved here?  The FE is claimed to be infinite in extent - so the mass must be infinite - doesn't this mean that the acceleration must be zero?!?

2) It says: "its effect on smaller bodies is negligible (similar to gravity in RET cosmology, which only has a noticeable affect on very large objects)."  But in RET, gravity is a force between two objects.  A pebble pulls on the Earth with the exact same force that the Earth pulls on the pebble...Newton's Third Law guys.  The reason the pebble accelerates faster than the Earth is because it's so much lighter and F=ma, so: a=F/m.  So what's the rule in FET?

3) "The gravitational pull of the stars, for example, causes observable tidal effects on Earth."  So could you please explain to me why the tides vary in timing from day to day when the stars are always in the same exact position at any given time of day?  Where do the stars go to when there is a low tide at midnight?  I can still see them.  Did their gravity gravitation turn off for some reason?  Why are there two high and two low tides each day.  This is not explained anywhere on the Wiki.

4) "Q: Why does gravity vary with altitude?  A: The moon and stars have a slight gravitational pull."  How doe this "gravitation" (but not "gravity" - even though you just called it that by mistake!) produce both a variable attraction for the tides and a constant attraction for reduced gravity on mountain tops?

Then there is another page https://wiki.tfes.org/Celestial_Gravitation ...

5) It says "Celestial Gravitation accounts for tides and other gravimetric anomalies across the Earth's plane."   How exactly does it "account for it"?  This isn't explained anywhere I could find.

Thanks in advance for your help!  (You know, you should really stick to saying "Just read the Wiki" and "False" and let Tom take the hits for the gaping holes in FET.)

Excellent quote mining, but if you actually go back and read the whole post of mine you pulled that excerpt from, you will notice that your wall of text here isn't even remotely related to what I said. However, my answer for you remains the same even if you keep ignoring it. If you are struggling this hard with the wiki, I honestly don't know how much I can help you. I know RE logicians have a rough time with basic logic and reading comprehension on a regular basis, so what you are exhibiting is nothing new. The best I can do for you is to suggest you head over to rif.org and take a look at the resources they offer. Once you have a better understanding, I encourage you to come back and try again after actually making an attempt. I wish you the best of luck in this endeavor.
Title: Re: Why is the Earth accelerating at9.8 metres per second?
Post by: Rama Set on September 28, 2017, 02:45:09 PM
There are a number of problems with UA.  If it is a proper acceleration, what is the source of energy that powers it?  If it is a coordinate acceleration, then what mechanism is causing the FoR of the Earth to move differently than another FoR, and how is anyone even sure that this other FoR exists?  How does one distinguish UA from GR?  How does UA explain the gravitational attraction of mountains?  The Cavendish experiment?  Gravity waves?  As previously mentioned, the gravitational effects of celestial bodies has multiple properties that manifest based on what you are observing.  How does anyone know that celestial bodies have gravitation?
Title: Re: Why is the Earth accelerating at9.8 metres per second?
Post by: Ga_x2 on September 28, 2017, 02:56:58 PM
I hope you are not an elementary school teacher.
I am not, but I sure feel like one after that exchange. Although, I imagine elementary students wouldn't struggle as much as you have here. I suppose I would be a remedial elementary teacher in this case.
not doing his job. Again: what is not easy to observe? You are avoiding the issue.
Title: Re: Why is the Earth accelerating at9.8 metres per second?
Post by: 3DGeek on September 28, 2017, 02:58:53 PM
I hope you are not an elementary school teacher.
I am not, but I sure feel like one after that exchange. Although, I imagine elementary students wouldn't struggle as much as you have here. I suppose I would be a remedial elementary teacher in this case.


Feel free to go and read it again. If you find yourself struggling to find it, just ask and I will do my best to help you.

Oh!  Cool!  So I've read the Wiki and I'm definitely struggling with it - so I'm asking you to do your best to help me.  Evidently I just needed to do the "Jeopardy!" thing where I phrase my answers in the form of a question!

So I just read this page: https://wiki.tfes.org/Universal_Acceleration and I'm struggling with it:

1) It says: "This force is known as "Universal Acceleration"" - how can a force be an acceleration?   Force is defined as acceleration times mass.  Which mass is involved here?  The FE is claimed to be infinite in extent - so the mass must be infinite - doesn't this mean that the acceleration must be zero?!?

2) It says: "its effect on smaller bodies is negligible (similar to gravity in RET cosmology, which only has a noticeable affect on very large objects)."  But in RET, gravity is a force between two objects.  A pebble pulls on the Earth with the exact same force that the Earth pulls on the pebble...Newton's Third Law guys.  The reason the pebble accelerates faster than the Earth is because it's so much lighter and F=ma, so: a=F/m.  So what's the rule in FET?

3) "The gravitational pull of the stars, for example, causes observable tidal effects on Earth."  So could you please explain to me why the tides vary in timing from day to day when the stars are always in the same exact position at any given time of day?  Where do the stars go to when there is a low tide at midnight?  I can still see them.  Did their gravity gravitation turn off for some reason?  Why are there two high and two low tides each day.  This is not explained anywhere on the Wiki.

4) "Q: Why does gravity vary with altitude?  A: The moon and stars have a slight gravitational pull."  How doe this "gravitation" (but not "gravity" - even though you just called it that by mistake!) produce both a variable attraction for the tides and a constant attraction for reduced gravity on mountain tops?

Then there is another page https://wiki.tfes.org/Celestial_Gravitation ...

5) It says "Celestial Gravitation accounts for tides and other gravimetric anomalies across the Earth's plane."   How exactly does it "account for it"?  This isn't explained anywhere I could find.

Thanks in advance for your help!  (You know, you should really stick to saying "Just read the Wiki" and "False" and let Tom take the hits for the gaping holes in FET.)

Excellent quote mining, but if you actually go back and read the whole post of mine you pulled that excerpt from, you will notice that your wall of text here isn't even remotely related to what I said. However, my answer for you remains the same even if you keep ignoring it. If you are struggling this hard with the wiki, I honestly don't know how much I can help you. I know RE logicians have a rough time with basic logic and reading comprehension on a regular basis, so what you are exhibiting is nothing new. The best I can do for you is to suggest you head over to rif.org and take a look at the resources they offer. Once you have a better understanding, I encourage you to come back and try again after actually making an attempt. I wish you the best of luck in this endeavor.

Oh good grief.  Is that the best you can do?   Let me translate your post into English:  "I can't answer a single one of your questions even though I said I would - so I'm going to claim that you're an idiot."

Fine...just fine...this is what we all expect of FE'ers.  Tom does the exact same thing.  Wait until the going gets tough, then deflect and bail...never, EVER engage in the discussion when it looks like you're going to lose.

It's just pathetic.  Man-up dude.  Either provide clear answers these entirely reasonable and on-topic questions about the Wiki articles relating to the question at hand - or admit that you can't.  (or just deflect/evade/bail as usual and we'll just assume the latter).
Title: Re: Why is the Earth accelerating at9.8 metres per second?
Post by: juner on September 28, 2017, 03:06:21 PM
not doing his job.
I agree. Based on your struggles so far I am playing the role of a remedial elementary teacher who is clearly not doing a good job because you still are struggling with something so simple.

Again: what is not easy to observe? You are avoiding the issue.
What is there to avoid? All I did was ask a question you still haven't answered. I am not sure why it is so difficult for you.


Oh good grief.  Is that the best you can do?   
I am sorry if pointing out simple observations isn't good enough for you.

Let me translate your post into English:
Are you suggesting it wasn't in English?

"I can't answer a single one of your questions even though I said I would - so I'm going to claim that you're an idiot."
Where did I say I would do that? Did you actually go back and read the post like I told you to? Clearly, you did not, or you wouldn't be having such a hard time with this.

Fine...just fine...this is what we all expect of FE'ers.  Tom does the exact same thing.  Wait until the going gets tough, then deflect and bail...never, EVER engage in the discussion when it looks like you're going to lose.
Oh hey, another rant when 3D doesn't get what he wants after posting a wall of text in response to something he didn't understand.

It's just pathetic.  Man-up dude.
Did you just assume my gender?

Either provide clear answers these entirely reasonable and on-topic questions about the Wiki articles relating to the question at hand - or admit that you can't.  (or just deflect/evade/bail as usual and we'll just assume the latter).
You can assume whatever you would like. Turns out that you are not the arbiter of the thread. All you did was quote-mine something I said, then posted a wall of questions that had no bearing on what I said. That isn't how conversation works, friend.
Title: Re: Why is the Earth accelerating at9.8 metres per second?
Post by: Ga_x2 on September 28, 2017, 03:16:17 PM
Again: what is not easy to observe? You are avoiding the issue.
What is there to avoid? All I did was ask a question you still haven't answered. I am not sure why it is so difficult for you.
ok. To my hinting at the cavendish experiment, you asked:
Quote
You think that is more easily observed than acceleration?
And this is the third time I'm asking you in response: what is difficult to observe?
And you are avoiding to get into the meat of the argument, instead concentrating on asperger level pedantries.
Title: Re: Why is the Earth accelerating at9.8 metres per second?
Post by: juner on September 28, 2017, 03:19:51 PM
Again: what is not easy to observe? You are avoiding the issue.
What is there to avoid? All I did was ask a question you still haven't answered. I am not sure why it is so difficult for you.
ok. To my hinting at the cavendish experiment, you asked:
Quote
You think that is more easily observed than acceleration?
And this is the third time I'm asking you in response: what is difficult to observe?
And you are avoiding to get into the meat of the argument, instead concentrating on asperger level pedantries.
So, I get that words are apparently hard for you, but I didn't actually say anything was difficult to observe. Feel free to show where I said anything to the contrary, and I will retract my claim. All I did was ask you a question which you are still avoiding for some reason.
Title: Re: Why is the Earth accelerating at9.8 metres per second?
Post by: Ga_x2 on September 28, 2017, 03:36:28 PM
Again: what is not easy to observe? You are avoiding the issue.
What is there to avoid? All I did was ask a question you still haven't answered. I am not sure why it is so difficult for you.
ok. To my hinting at the cavendish experiment, you asked:
Quote
You think that is more easily observed than acceleration?
And this is the third time I'm asking you in response: what is difficult to observe?
And you are avoiding to get into the meat of the argument, instead concentrating on asperger level pedantries.
So, I get that words are apparently hard for you, but I didn't actually say anything was difficult to observe. Feel free to show where I said anything to the contrary, and I will retract my claim. All I did was ask you a question which you are still avoiding for some reason.
I rest my case. Holy faq you have taken hairsplitting to a artform.
So.
When you say:
Quote
You think that is more easily observed than acceleration?

what does that "that" there stands for?
Title: Re: Why is the Earth accelerating at9.8 metres per second?
Post by: Curious Squirrel on September 28, 2017, 03:37:44 PM
Again: what is not easy to observe? You are avoiding the issue.
What is there to avoid? All I did was ask a question you still haven't answered. I am not sure why it is so difficult for you.
ok. To my hinting at the cavendish experiment, you asked:
Quote
You think that is more easily observed than acceleration?
And this is the third time I'm asking you in response: what is difficult to observe?
And you are avoiding to get into the meat of the argument, instead concentrating on asperger level pedantries.
Don't bother engaging with Junker. He'll quote mine, ignore context, and play word games until he can twist things to make himself feel he's won. It's not worth it. He does a fine job moderating (even if I feel he's a bit biased) but he doesn't actually discuss. He just sits on his high horse.
Title: Re: Why is the Earth accelerating at9.8 metres per second?
Post by: juner on September 28, 2017, 03:41:33 PM
When you say:
Quote
You think that is more easily observed than acceleration?

what does that "that" there stands for?
It means do you Ga_x2 think "that" Cavendish/gravity is more easily observed than acceleration?


Don't bother engaging with Junker. He'll quote mine, ignore context, and play word games until he can twist things to make himself feel he's won. It's not worth it. He does a fine job moderating (even if I feel he's a bit biased) but he doesn't actually discuss. He just sits on his high horse.
Cool opinion. Sorry if I have upset you in the past. Turns out I do actually discuss, I just prefer to discuss the dumb things RE logicians say when they say them (see:frequently).
Title: Re: Why is the Earth accelerating at9.8 metres per second?
Post by: Ga_x2 on September 28, 2017, 03:57:10 PM
When you say:
Quote
You think that is more easily observed than acceleration?

what does that "that" there stands for?
It means do you Ga_x2 think "that" Cavendish/gravity is more easily observed than acceleration?
if I were your horse I'd be high as a kyte too. My God why did we have to go through all that to get to this.

So.
When you drop something to the floor, the only immediate observation is that the ball accelerates toward it. To go from there to "there is something that attracts stuff and that attraction is proportional to the mass and inversely proportional to the distance and I'm calling this thing gravity" is a stretch.

The Cavendish experiment shows that there is something like what I've described above. The immediate observation is that the balls accelerate toward each other, but you have nothing clearly acting on them, and the edit: attraction is dependent on mass and distance. My understanding is that we call this thing gravity, even if we don't yet know the underpinnings. So the short answer is yes. Why do you think otherwise?
Title: Re: Why is the Earth accelerating at9.8 metres per second?
Post by: juner on September 28, 2017, 04:03:10 PM
So.
When you drop something to the floor, the only immediate observation is that the ball accelerates toward it.


To go from there to "there is something that attracts stuff and that attraction is proportional to the mass and inversely proportional to the distance and I'm calling this thing gravity" is a stretch.

The Cavendish experiment shows that there is something like what I've described above. The immediate observation is that the balls accelerate toward each other, but you have nothing clearly acting on them, and the acceleration is dependent on mass and distance. My understanding is that we call this thing gravity, even if we don't yet know the underpinnings. So the short answer is yes. Why do you think otherwise?

I see, so you think observing the Cavendish experiment is easier than observing acceleration. That was a long-winded reply when you could have just said yes and saved two pages of replies. I mean, you are objectively wrong, but at least you gave a sort-of-answer finally.
Title: Re: Why is the Earth accelerating at9.8 metres per second?
Post by: Ga_x2 on September 28, 2017, 04:10:14 PM
So.
When you drop something to the floor, the only immediate observation is that the ball accelerates toward it.


To go from there to "there is something that attracts stuff and that attraction is proportional to the mass and inversely proportional to the distance and I'm calling this thing gravity" is a stretch.

The Cavendish experiment shows that there is something like what I've described above. The immediate observation is that the balls accelerate toward each other, but you have nothing clearly acting on them, and the acceleration is dependent on mass and distance. My understanding is that we call this thing gravity, even if we don't yet know the underpinnings. So the short answer is yes. Why do you think otherwise?

I see, so you think observing the Cavendish experiment is easier than observing acceleration. That was a long-winded reply when you could have just said yes and saved two pages of replies. I mean, you are objectively wrong, but at least you gave a sort-of-answer finally.
what's wrong with my answer?
Title: Re: Why is the Earth accelerating at9.8 metres per second?
Post by: juner on September 28, 2017, 04:12:34 PM
what's wrong with my answer?

It is incorrect. That is what is wrong with it.
Title: Re: Why is the Earth accelerating at9.8 metres per second?
Post by: Ga_x2 on September 28, 2017, 04:22:13 PM
what's wrong with my answer?

It is incorrect. That is what is wrong with it.
you are not really interested in having a conversation are you? Or are you just having a blast exploiting my good nature?
Why is it incorrect?
Title: Re: Why is the Earth accelerating at9.8 metres per second?
Post by: juner on September 28, 2017, 04:25:24 PM
you are not really interested in having a conversation are you?
Of course I am.


Why is it incorrect?
Because observing acceleration is objectively easier than observing gravity.
Title: Re: Why is the Earth accelerating at9.8 metres per second?
Post by: Curious Squirrel on September 28, 2017, 04:30:32 PM
you are not really interested in having a conversation are you?
Of course I am.
Incorrect

Why is it incorrect?
Because observing acceleration is objectively easier than observing gravity.
False
Title: Re: Why is the Earth accelerating at9.8 metres per second?
Post by: juner on September 28, 2017, 04:37:47 PM
Because observing acceleration is objectively easier than observing gravity.
False

I mean, I know RE logicians can be kind of dense, but do you actually believe this? Do you understand what acceleration is? I honestly can't believe this discussion is happening right now (or that I am replying to it).

Title: Re: Why is the Earth accelerating at9.8 metres per second?
Post by: Ga_x2 on September 28, 2017, 04:41:26 PM
you are not really interested in having a conversation are you?
Of course I am.


Why is it incorrect?
Because observing acceleration is objectively easier than observing gravity.
then you missed entirely the sense of my quip about Cavendish! Of course it is, that's trivially true. Why would you ever ask me that? Just to enjoy 3 pages of me trying to understand what the heck you were  hinting at?

I thought (wrongly) you were referring to the difficulty of observing gravity in the context of UA vs. The context of a lab experiment. My bad. Now I know it. You have literally no depth in your comments. I'll never try and read anything else beyond what's clearly stated. Also because you're not even trying to narrow the gap in the misunderstandings,
 because that's your way of having fun.

I take everything back and start afresh. Do you have any opinion on the Cavendish experiment? Would you mind sharing it?
Title: Re: Why is the Earth accelerating at9.8 metres per second?
Post by: juner on September 28, 2017, 04:45:55 PM
Why would you ever ask me that? Just to enjoy 3 pages of me trying to understand what the heck you were  hinting at?
Because I was looking for a simple yes or no based on your one-word reply with an emoticon. I am not sure why you read so much into it.

I thought (wrongly) you were referring to the difficulty of observing gravity in the context of UA vs. The context of a lab experiment. My bad. Now I know it. You have literally no depth in your comments.
I really try to keep it simple for RE logicians. Turns out you still take it in any direction you want to...

I'll never try and read anything else beyond what's clearly stated.
I am not sure why you would do otherwise.

Also because you're not even trying to narrow the gap in the misunderstandings,
because that's your way of having fun.
I apologize if a short question caused you a misunderstanding. I assure you it isn't fun for me.

I take everything back and start afresh. Do you have any opinion on the Cavendish experiment?
Not really.

Would you mind sharing it?
Sure, I suppose. Preferably in a topic that has something to do with it.
Title: Re: Why is the Earth accelerating at9.8 metres per second?
Post by: Ga_x2 on September 28, 2017, 04:57:11 PM
Why would you ever ask me that? Just to enjoy 3 pages of me trying to understand what the heck you were  hinting at?
Because I was looking for a simple yes or no based on your one-word reply with an emoticon. I am not sure why you read so much into it.
I overestimated your interest in having a conversation about topics instead of a metaconversation about words.

Quote
I assure you it isn't fun for me.
False.
 ;D

Quote
Sure, I suppose. Preferably in a topic that has something to do with it.
It has A LOT to do with the OP.

Now keeping it in vein, you saying that you haven't an opinion would mean that you have nothing to share...
Title: Re: Why is the Earth accelerating at9.8 metres per second?
Post by: juner on September 28, 2017, 04:58:58 PM
Now keeping it in vein, you saying that you haven't an opinion would mean that you have nothing to share...
I didn't say I don't have an opinion. You are already doing the thing you said you would stop doing. Oh well.
Title: Re: Why is the Earth accelerating at9.8 metres per second?
Post by: Ga_x2 on September 28, 2017, 05:23:28 PM
Now keeping it in vein, you saying that you haven't an opinion would mean that you have nothing to share...
I didn't say I don't have an opinion. You are already doing the thing you said you would stop doing. Oh well.
Shoot!  :-X  ;D
Chalk this up to English not being my native language... I read "not really" as meaning absolutely not.
So you do have an opinion, sorta, ok good.
So. Back again: it has A LOT to do with the OP.
Please share
Title: Re: Why is the Earth accelerating at9.8 metres per second?
Post by: juner on September 28, 2017, 05:28:32 PM
So. Back again: it has A LOT to do with the OP.
Please share

It actually doesn't have anything to do with the OP. Are you sure you are in the correct thread? OP is actually, literally specific about acceleration.
Title: Re: Why is the Earth accelerating at9.8 metres per second?
Post by: Ga_x2 on September 28, 2017, 05:35:25 PM
So. Back again: it has A LOT to do with the OP.
Please share

It actually doesn't have anything to do with the OP. Are you sure you are in the correct thread? OP is actually, literally specific about acceleration.
see you're probably not getting this because of your kinda sorta opinion about ye old Cavendish. Share, and then we'll have some foundation to build on  :)
Title: Re: Why is the Earth accelerating at9.8 metres per second?
Post by: Curious Squirrel on September 28, 2017, 05:36:40 PM
Because observing acceleration is objectively easier than observing gravity.
False

I mean, I know RE logicians can be kind of dense, but do you actually believe this? Do you understand what acceleration is? I honestly can't believe this discussion is happening right now (or that I am replying to it).
Hold out an object above the ground. Let go. Bam, observing gravity. ;)
Title: Re: Why is the Earth accelerating at9.8 metres per second?
Post by: juner on September 28, 2017, 05:59:26 PM
see you're probably not getting this because of your kinda sorta opinion about ye old Cavendish. Share, and then we'll have some foundation to build on  :)
You can continue to deflect as much as you'd like when it is pointed out that you are wrong, but it doesn't make you any less wrong. I figured you would have learned that by now.


Hold out an object above the ground. Let go. Bam, observing gravity. ;)
Oh cool. How do you measure the "gravity" you are observing in your example?
Title: Re: Why is the Earth accelerating at9.8 metres per second?
Post by: Ga_x2 on September 28, 2017, 06:32:24 PM
see you're probably not getting this because of your kinda sorta opinion about ye old Cavendish. Share, and then we'll have some foundation to build on  :)
You can continue to deflect as much as you'd like when it is pointed out that you are wrong, but it doesn't make you any less wrong. I figured you would have learned that by now.
I'm sorry, this time it's you the one not getting it, but I don't want to beat around the bushes for hours on end.
Tell you what, if you don't feel like discussing this here, comment on the question I posed as one of my first posts in the Q&A forum, if you really are interested in a conversation. Or at least in trying to provide an explanation for a plain observation, which has, again, A LOT to do with the value of the UA the OP is quoting. We can always come back here later.
It's easy to find, it's the one with 0 replies. I can resurrect it for you, too.
Title: Re: Why is the Earth accelerating at9.8 metres per second?
Post by: 3DGeek on September 28, 2017, 06:42:19 PM
So.
When you drop something to the floor, the only immediate observation is that the ball accelerates toward it. To go from there to "there is something that attracts stuff and that attraction is proportional to the mass and inversely proportional to the distance and I'm calling this thing gravity" is a stretch.

Yeah - but that's what Leonardo Da Vinci did.   Newton, however, used the same basic principle (stuff falls, it accelerates as it does so, the acceleration rate is independent of mass) - and applied it to the motion of planets, the moon, everything else he could get data for.   That's how HE figured out how gravity is proportional to the product of the two masses divided by the square of the distance between them.

So the FET "universal accelerator" would certainly have fooled Da Vinci - but would not have persuaded Sir Isaac Newton.

Quote
The Cavendish experiment shows that there is something like what I've described above. The immediate observation is that the balls accelerate toward each other, but you have nothing clearly acting on them, and the edit: attraction is dependent on mass and distance. My understanding is that we call this thing gravity, even if we don't yet know the underpinnings. So the short answer is yes. Why do you think otherwise?

The Cavendish experiment was really an effort to calculate the constant in Newton's Law of Gravity. ("Big G")   You can't easily work it out without knowing the masses of the objects involved - which is hard to figure out for stars, planets, moons, etc.  Hence Cavendish (and others who refined his experiment) use known masses so they can figure out 'G' directly rather than from 'g and guesstimated masses for planets and such.   We don't need Cavendish to explain how gravity works - that can be deduced by watching the motion of planets in a heliocentric solar system.


Title: Re: Why is the Earth accelerating at9.8 metres per second?
Post by: Ga_x2 on September 28, 2017, 06:50:19 PM
The Cavendish experiment was really an effort to calculate the constant in Newton's Law of Gravity. ("Big G")   You can't easily work it out without knowing the masses of the objects involved - which is hard to figure out for stars, planets, moons, etc.  Hence Cavendish (and others who refined his experiment) use known masses so they can figure out 'G' directly rather than from 'g and guesstimated masses for planets and such.   We don't need Cavendish to explain how gravity works - that can be deduced by watching the motion of planets in a heliocentric solar system.
Sure, but that's not why it's important in this context. In this thread we are not anywhere near any G yet, big or small. We could go faster, but the people here love their hair thoroughly splitted.
EDIT: you work with too many assumptions. Newton assumed a 'RE. "No shit sherlock" you are probably saying, but that's precisely the problem ;D
Title: Re: Why is the Earth accelerating at9.8 metres per second?
Post by: juner on September 28, 2017, 07:02:31 PM
It's easy to find, it's the one with 0 replies.
Must not have been a very interesting topic, then.
Title: Re: Why is the Earth accelerating at9.8 metres per second?
Post by: Ga_x2 on September 28, 2017, 07:04:46 PM
It's easy to find, it's the one with 0 replies.
Must not have been a very interesting topic, then.
come on now you're overdoing it
 ;D
Edited to add: please refrain from low content posts in the upper fora :P
Title: Re: Why is the Earth accelerating at9.8 metres per second?
Post by: 3DGeek on September 28, 2017, 07:09:41 PM
The Cavendish experiment was really an effort to calculate the constant in Newton's Law of Gravity. ("Big G")   You can't easily work it out without knowing the masses of the objects involved - which is hard to figure out for stars, planets, moons, etc.  Hence Cavendish (and others who refined his experiment) use known masses so they can figure out 'G' directly rather than from 'g and guesstimated masses for planets and such.   We don't need Cavendish to explain how gravity works - that can be deduced by watching the motion of planets in a heliocentric solar system.
Sure, but that's not why it's important in this context. In this thread we are not anywhere near any G yet, big or small. We could go faster, but the people here love their hair thoroughly splitted.
EDIT: you work with too many assumptions. Newton assumed a 'RE. "No shit sherlock" you are probably saying, but that's precisely the problem ;D

Well, he famously said: "If I have seen further, it is by standing on the shoulders of giants."

He stood on the shoulders of Da Vinci, Galileo, Copernicus and others.  They proved the heliocentric model...which depends on the earth being round.

But by the 1650's there had been enough circumnavigations and other long distance ocean travel using only "star and compass" navigation that any rational doubt as to the shape and size of the earth had been completely answered.

It's entirely unreasonable to expect every new generation of scientists to re-prove things that are already well established...just as you no longer have to convince yourself that apples are not poisonous before you eat one.  There are things that are just so well established that they can't be wrong.

Unless, of course, you're a Flat Earther.

Title: Re: Why is the Earth accelerating at9.8 metres per second?
Post by: juner on September 28, 2017, 07:31:44 PM
It's easy to find, it's the one with 0 replies.
Must not have been a very interesting topic, then.
come on now you're overdoing it
 ;D
Edited to add: please refrain from low content posts in the upper fora :P

I mean, I could have easily warned you for your initial low-content post that initiated the whole mess, but I didn't. However, seeing as you are not a moderator, I will kindly ask that you don't attempt to moderate threads. This can go ahead and be your last freebie.
Title: Re: Why is the Earth accelerating at9.8 metres per second?
Post by: Ga_x2 on September 28, 2017, 07:49:02 PM
It's easy to find, it's the one with 0 replies.
Must not have been a very interesting topic, then.
come on now you're overdoing it
 ;D
Edited to add: please refrain from low content posts in the upper fora :P

I mean, I could have easily warned you for your initial low-content post that initiated the whole mess, but I didn't. However, seeing as you are not a moderator, I will kindly ask that you don't attempt to moderate threads. This can go ahead and be your last freebie.
*Adding humour to the list of things not to use with you.
So are you willing to discuss cavendish here or elsewhere or shall I put that on the list of things FE proponents seem not to have an opinion/understand/care to discuss?
Title: Re: Why is the Earth accelerating at9.8 metres per second?
Post by: juner on September 28, 2017, 08:01:09 PM
*Adding humour to the list of things not to use with you.
You certainly can use humor. I would even suggest trying sometime.

So are you willing to discuss cavendish here or elsewhere or shall I put that on the list of things FE proponents seem not to have an opinion/understand/care to discuss?
Why would I discuss Cavendish in a thread that has nothing to do with it? I know you RE proponents think this place exist for your personal whims, but I assure you that is not the case. While it certainly has been fun and hopefully you have finally learned something, I will ask that your next post in this thread try to stay on topic. It is common for RE logicians to resort to changing subjects, "whataboutisms," and moving the goal posts when they are shown to be wrong during discussion, so I will ask you not to do that here.
Title: Re: Why is the Earth accelerating at9.8 metres per second?
Post by: Rama Set on September 28, 2017, 08:03:27 PM
Considering that the force of gravity on Earth is tied to the gravitational constant, the measuring of this constant would be extremely relevant.
Title: Re: Why is the Earth accelerating at9.8 metres per second?
Post by: juner on September 28, 2017, 08:06:18 PM
Considering that the force of gravity on Earth is tied to the gravitational constant, the measuring of this constant would be extremely relevant.

It would be relevant in a thread about gravity, the gravitational constant, or measuring said constant. Oddly enough, not a single one of those things are mentioned in the OP. You are welcome to make a thread about that topic if you'd like.
Title: Re: Why is the Earth accelerating at9.8 metres per second?
Post by: Ga_x2 on September 28, 2017, 08:06:56 PM

So are you willing to discuss cavendish here or elsewhere or shall I put that on the list of things FE proponents seem not to have an opinion/understand/care to discuss?
Why would I discuss Cavendish in a thread that has nothing to do with it?
you are still not getting why it's relevant with the concept of UA of the OP.
Fine. Ubi major minor cessat. Maybe when you feel like making an effort or contributing something substantial resurrect the other thread. Besos
Title: Re: Why is the Earth accelerating at9.8 metres per second?
Post by: Rama Set on September 28, 2017, 08:12:00 PM
Considering that the force of gravity on Earth is tied to the gravitational constant, the measuring of this constant would be extremely relevant.

It would be relevant in a thread about gravity, the gravitational constant, or measuring said constant. Oddly enough, not a single one of those things are mentioned in the OP. You are welcome to make a thread about that topic if you'd like.

Come now, we all know that not every post must connect directly to the OP, especially in the debate forum. The Cavendish experiment is an excellent falsification of UA since it can measure an attraction between masses that is not due to electromagnetism. Variations have shown attraction perpendicular to the supposed vector of UA in precisely the amount predicted by GR and Newton's theory.
Title: Re: Why is the Earth accelerating at9.8 metres per second?
Post by: juner on September 28, 2017, 08:17:00 PM
Come now, we all know that not every post must connect directly to the OP, especially in the debate forum.
Most threads deviate in some manner (thanks to RE behaviors mentioned above), but this really is a separate topic.

The Cavendish experiment is an excellent falsification of UA since it can measure an attraction between masses that is not due to electromagnetism. Variations have shown attraction perpendicular to the supposed vector of UA in precisely the amount predicted by GR and Newton's theory.
If this was a thread about the flaws in UA, falsifying UA, or exploring the Cavendish experiment, then it would be worth discussing, along with other flaws. It is time to get the thread back on track. That is the great thing about these fora, anyone can make a topic to discuss instead of hijacking a thread on another topic.
Title: Re: Why is the Earth accelerating at9.8 metres per second?
Post by: Rama Set on September 28, 2017, 08:25:58 PM
In a debate, if the FE response to the OP is "UA ftw!", rebutting with, "Nyah! Nyah! Cavendish!" is appropriate. It's much more on topic than "observing acceleration is objectively easier than observing gravity." Although that statement is true, it doesn't change the fact that gravity has been observed on small and large scales.
Title: Re: Why is the Earth accelerating at9.8 metres per second?
Post by: juner on September 28, 2017, 08:31:10 PM
In a debate, if the FE response to the OP is "UA ftw!", rebutting with, "Nyah! Nyah! Cavendish!" is appropriate. It's much more on topic than "observing acceleration is objectively easier than observing gravity." Although that statement is true, it doesn't change the fact that gravity has been observed on small and large scales.

Except that the rebuttal was what actually happened, whereas you are meme-ifying the "FE response." I also disagree that it is "much more on topic." But I acknowledge I shouldn't have taken the RE bait and even bothered engaging. I should have just warned those posting and got the thread back on track at that time. But, it is done and I am getting it back on track now. So, if you have anything on the topic you want to contribute, feel free to do so. If you want to discuss another topic, feel free to make a new thread.
Title: Re: Why is the Earth accelerating at9.8 metres per second?
Post by: Rama Set on September 28, 2017, 08:39:32 PM
Fair game. The Cavendish Experiment shows UA to be a big bag of baloney. FE theory should really explore a modified gravity theory where the particles that make up the Earth are a heretofore Uncatalogued type not captured in the standard model and are gravitationally inactive. It makes much more sense than UA.
Title: Re: Why is the Earth accelerating at9.8 metres per second?
Post by: TomInAustin on September 28, 2017, 10:01:58 PM
I have read the Wiki and FAQ; (UA asserts that the Earth is accelerating 'upward' at a constant rate of 9.8m/s^2)

But am struggling to find either a reason why or proof that the earth IS accelerating at 9.8 metres per second:

Wiki states: "The are several explanations for UA. As it is difficult for proponents of Flat Earth Theory to obtain grant money for scientific research, it is nigh on impossible to determine which of these theories is correct."

I was hoping that a FE could explain why this theory exists without this reasoning and evidence? Why does it make sense to you and how did this exact speed come about? As for the several explanations for UA; what are they please as I could not see them listed anywhere?

many thanks

Has there been an answer to the OP?   Why is the Earth accelerating at 9.8 metres per second? 
Title: Re: Why is the Earth accelerating at9.8 metres per second?
Post by: Ga_x2 on September 28, 2017, 10:35:56 PM
I have read the Wiki and FAQ; (UA asserts that the Earth is accelerating 'upward' at a constant rate of 9.8m/s^2)

But am struggling to find either a reason why or proof that the earth IS accelerating at 9.8 metres per second:

Wiki states: "The are several explanations for UA. As it is difficult for proponents of Flat Earth Theory to obtain grant money for scientific research, it is nigh on impossible to determine which of these theories is correct."

I was hoping that a FE could explain why this theory exists without this reasoning and evidence? Why does it make sense to you and how did this exact speed come about? As for the several explanations for UA; what are they please as I could not see them listed anywhere?

many thanks

Has there been an answer to the OP?   Why is the Earth accelerating at 9.8 metres per second?
tbh the first part of the question is kind of a non issue. FE proponents can simply say it's a basic property of the universe, just as I understand gravity is. The second part, where he asks how you get that number and the reasoning behind it, hasn't been answered in detail, beside stating that's indistinguishable from gravity. But you're starting from the assumption that the A in Q&A stands for answer.
Title: Re: Why is the Earth accelerating at9.8 metres per second?
Post by: 3DGeek on September 29, 2017, 04:36:19 PM
The topic of the thread is "Why is the Earth accelerating at 9.8 metres per second?" - it's VERY clear (to both FE'ers and RE'ers that the premise of the question is flawed).

So to answer it in any useful way, it's really necessary to point out that:

a) It's not doing that (because: tides, Cavendish, equatorial-vs-poles, mountain-tops, etc).
b) Even if it was, then there would need to be an infinite energy source to power it (because: F=ma, W=Fs and E=W) - which is (at best) implausible.

So either the answer is either "It isn't" or "UA is not a viable hypothesis to explain it"...and that's the thing we need to discuss in order to fully answer the question.

If you're going to ban topic derailment - then it would be intellectually honest to ban a bunch of Tom's derailments...he's the king of derailment...and quite a few of your own.

So if there is a clear policy that all answers that do not DIRECTLY respond to the precise question are inappropriate - then so be it - but I'd hope you'd stick by that rule when I point out both your and Tom's infractions of it.

IMHO, that's not the policy I've seen enforced here.  For rational debate to proceed, a certain measure of topic-wandering is inevitable and not entirely undesirable.
Title: Re: Why is the Earth accelerating at9.8 metres per second?
Post by: juner on September 29, 2017, 05:02:46 PM
a) It's not doing that (because: tides, Cavendish, equatorial-vs-poles, mountain-tops, etc).
All of which are addressed in the wiki/FAQ and in various threads on these very fora. You may disagree, or they may be flawed and you may want to debate them, but it doesn't change the fact that they exist. This thread initially started in Q&A. I know it is hard for round earth logicians to read forum descriptions, but what this thread quickly changed into is not appropriate for Q&A. It is for FE questions and answers, not for RE to point out something they disagree with. Instead of locking the thread, I moved it here (which was apparently a mistake). I suppose I can just be more strict about enforcing rules, but then many of you will complain about that, too.


b) Even if it was, then there would need to be an infinite energy source to power it
False.

If you're going to ban topic derailment - then it would be intellectually honest to ban a bunch of Tom's derailments...he's the king of derailment...and quite a few of your own.
If I see them, or they get reported, then sure. You are also quite good at derailing yourself.

So if there is a clear policy that all answers that do not DIRECTLY respond to the precise question are inappropriate - then so be it - but I'd hope you'd stick by that rule when I point out both your and Tom's infractions of it.
No one ever said that was the policy. I'd suggest you stop building a strawman to continue your soapbox.

IMHO, that's not the policy I've seen enforced here.  For rational debate to proceed, a certain measure of topic-wandering is inevitable and not entirely undesirable.
Yes, and that is allowed to an extent. It is also apparent to anyone who has posted here for a while.
Title: Re: Why is the Earth accelerating at9.8 metres per second?
Post by: TomInAustin on September 29, 2017, 05:17:24 PM
a) It's not doing that (because: tides, Cavendish, equatorial-vs-poles, mountain-tops, etc).
All of which are addressed in the wiki/FAQ and in various threads on these very fora. You may disagree, or they may be flawed and you may want to debate them, but it doesn't change the fact that they exist. This thread initially started in Q&A. I know it is hard for round earth logicians to read forum descriptions, but what this thread quickly changed into is not appropriate for Q&A. It is for FE questions and answers, not for RE to point out something they disagree with. Instead of locking the thread, I moved it here (which was apparently a mistake). I suppose I can just be more strict about enforcing rules, but then many of you will complain about that, too.


b) Even if it was, then there would need to be an infinite energy source to power it
False.

Please expand on your answer.  It seems to be that in infinite power source would have to be present or we risk the energy running out and the acceleration ends.

What is the source of this energy in your opinion?

Title: Re: Why is the Earth accelerating at9.8 metres per second?
Post by: Ga_x2 on September 29, 2017, 05:51:13 PM

a) It's not doing that (because: tides, Cavendish, equatorial-vs-poles, mountain-tops, etc).
Cavendish doesn't disprove UA. It simply demonstrate that there are factors that need to be taken into account when calculating the value of said UA. I.e. the second part of the question. But junker said it ain't on topic and who am I to protest. :P
Title: Re: Why is the Earth accelerating at9.8 metres per second?
Post by: 3DGeek on September 29, 2017, 06:01:34 PM
a) It's not doing that (because: tides, Cavendish, equatorial-vs-poles, mountain-tops, etc).
All of which are addressed in the wiki/FAQ and in various threads on these very fora. You may disagree, or they may be flawed and you may want to debate them, but it doesn't change the fact that they exist. This thread initially started in Q&A. I know it is hard for round earth logicians to read forum descriptions, but what this thread quickly changed into is not appropriate for Q&A. It is for FE questions and answers, not for RE to point out something they disagree with. Instead of locking the thread, I moved it here (which was apparently a mistake). I suppose I can just be more strict about enforcing rules, but then many of you will complain about that, too.


b) Even if it was, then there would need to be an infinite energy source to power it
False.

Please expand on your answer.  It seems to be that in infinite power source would have to be present or we risk the energy running out and the acceleration ends.

What is the source of this energy in your opinion?

Think about it though - Energy is force times distance through which it's applied.   Force is mass times acceleration.

So we know the acceleration - and we can calculate the distance (assuming the force has been applied at about the same rate at least throughout recorded history).

The other thing is the mass.   What is the mass of the Flat Earth?   Mass is volume times density.  We know the area of the Flat Earth is infinite (Yes, I read the Wiki) and the thickness is greater than zero - and the density is greater than zero...and therefore the force required is infinite and if the Earth moves by even one millimeter then the energy required to move it is also infinite.

Too many infinities.

Title: Re: Why is the Earth accelerating at9.8 metres per second?
Post by: TomInAustin on September 29, 2017, 08:04:52 PM
a) It's not doing that (because: tides, Cavendish, equatorial-vs-poles, mountain-tops, etc).
All of which are addressed in the wiki/FAQ and in various threads on these very fora. You may disagree, or they may be flawed and you may want to debate them, but it doesn't change the fact that they exist. This thread initially started in Q&A. I know it is hard for round earth logicians to read forum descriptions, but what this thread quickly changed into is not appropriate for Q&A. It is for FE questions and answers, not for RE to point out something they disagree with. Instead of locking the thread, I moved it here (which was apparently a mistake). I suppose I can just be more strict about enforcing rules, but then many of you will complain about that, too.


b) Even if it was, then there would need to be an infinite energy source to power it
False.

Please expand on your answer.  It seems to be that in infinite power source would have to be present or we risk the energy running out and the acceleration ends.

What is the source of this energy in your opinion?

Think about it though - Energy is force times distance through which it's applied.   Force is mass times acceleration.

So we know the acceleration - and we can calculate the distance (assuming the force has been applied at about the same rate at least throughout recorded history).

The other thing is the mass.   What is the mass of the Flat Earth?   Mass is volume times density.  We know the area of the Flat Earth is infinite (Yes, I read the Wiki) and the thickness is greater than zero - and the mass is greater than zero...and therefore the force required is infinite and if the Earth moves by even one millimeter then the energy required to move it is also infinite.

Too many infinities.

Good catch, it would have to be an infinite source of power.
Title: Re: Why is the Earth accelerating at9.8 metres per second?
Post by: Curious Squirrel on September 29, 2017, 08:09:39 PM
a) It's not doing that (because: tides, Cavendish, equatorial-vs-poles, mountain-tops, etc).
All of which are addressed in the wiki/FAQ and in various threads on these very fora. You may disagree, or they may be flawed and you may want to debate them, but it doesn't change the fact that they exist. This thread initially started in Q&A. I know it is hard for round earth logicians to read forum descriptions, but what this thread quickly changed into is not appropriate for Q&A. It is for FE questions and answers, not for RE to point out something they disagree with. Instead of locking the thread, I moved it here (which was apparently a mistake). I suppose I can just be more strict about enforcing rules, but then many of you will complain about that, too.


b) Even if it was, then there would need to be an infinite energy source to power it
False.

Please expand on your answer.  It seems to be that in infinite power source would have to be present or we risk the energy running out and the acceleration ends.

What is the source of this energy in your opinion?

Think about it though - Energy is force times distance through which it's applied.   Force is mass times acceleration.

So we know the acceleration - and we can calculate the distance (assuming the force has been applied at about the same rate at least throughout recorded history).

The other thing is the mass.   What is the mass of the Flat Earth?   Mass is volume times density.  We know the area of the Flat Earth is infinite (Yes, I read the Wiki) and the thickness is greater than zero - and the mass is greater than zero...and therefore the force required is infinite and if the Earth moves by even one millimeter then the energy required to move it is also infinite.

Too many infinities.

Good catch, it would have to be an infinite source of power.
Not quite. UA FE doesn't have a infinite plane (from my understanding) infinite plane FE uses normal gravity. UA only applies to FE's with finite dimension, though what those are are of course unknown.
Title: Re: Why is the Earth accelerating at9.8 metres per second?
Post by: JHelzer on September 29, 2017, 08:27:44 PM
Not quite. UA FE doesn't have a infinite plane (from my understanding) infinite plane FE uses normal gravity. UA only applies to FE's with finite dimension, though what those are are of course unknown.

I once read somewhere on here that if you got to the edge of the flat Earth and jumped off you might step outside Earth's shielding from UE.  That would mean that UE would start acting on you and you'd accelerate at 9.8 m/s along with the edge of the flat Earth right next to you.  That is an experiment someone should try.
Title: Re: Why is the Earth accelerating at9.8 metres per second?
Post by: 3DGeek on September 30, 2017, 03:20:38 PM
a) It's not doing that (because: tides, Cavendish, equatorial-vs-poles, mountain-tops, etc).
All of which are addressed in the wiki/FAQ and in various threads on these very fora. You may disagree, or they may be flawed and you may want to debate them, but it doesn't change the fact that they exist. This thread initially started in Q&A. I know it is hard for round earth logicians to read forum descriptions, but what this thread quickly changed into is not appropriate for Q&A. It is for FE questions and answers, not for RE to point out something they disagree with. Instead of locking the thread, I moved it here (which was apparently a mistake). I suppose I can just be more strict about enforcing rules, but then many of you will complain about that, too.


b) Even if it was, then there would need to be an infinite energy source to power it
False.

Please expand on your answer.  It seems to be that in infinite power source would have to be present or we risk the energy running out and the acceleration ends.

What is the source of this energy in your opinion?

Think about it though - Energy is force times distance through which it's applied.   Force is mass times acceleration.

So we know the acceleration - and we can calculate the distance (assuming the force has been applied at about the same rate at least throughout recorded history).

The other thing is the mass.   What is the mass of the Flat Earth?   Mass is volume times density.  We know the area of the Flat Earth is infinite (Yes, I read the Wiki) and the thickness is greater than zero - and the mass is greater than zero...and therefore the force required is infinite and if the Earth moves by even one millimeter then the energy required to move it is also infinite.

Too many infinities.

Good catch, it would have to be an infinite source of power.
Not quite. UA FE doesn't have a infinite plane (from my understanding) infinite plane FE uses normal gravity. UA only applies to FE's with finite dimension, though what those are are of course unknown.

That would be quite logical - but it's not what the Wiki says - and not what the FE'ers on this forum seem to be claiming.

There are four possibilities here:

* Infinite disk + UA has the "infinite energy" problem - but it's what most TFES.org people are saying.
* Infinite disk + Gravity would be an easier proposition to defend.
* Finite disk + UA is more plausible than with an infinite disk...although it's still not easy.
* Finite disk + Gravity really doesn't work because you'd be walking uphill anytime you're far from the center.
Title: Re: Why is the Earth accelerating at9.8 metres per second?
Post by: Pete Svarrior on September 30, 2017, 03:31:27 PM
* Infinite disk + UA has the "infinite energy" problem - but it's what most TFES.org people are saying.
Why must you insist on misrepresenting our position? No one is falling for it.
Title: Re: Why is the Earth accelerating at9.8 metres per second?
Post by: xenotolerance on September 30, 2017, 04:23:43 PM
No one is falling for you evading the issue, either. This is the point:

Quote
* Infinite disk + UA has the "infinite energy" problem.
* Infinite disk + Gravity would be an easier proposition to defend.
* Finite disk + UA is more plausible than with an infinite disk...although it's still not easy.
* Finite disk + Gravity really doesn't work because you'd be walking uphill anytime you're far from the center.

The wiki posits the third option, the finite disk moved by universal acceleration, but individual flat Earth believers variously believe in different things. The debate can split or otherwise adapt to any of these or other sub-topics.
Title: Re: Why is the Earth accelerating at9.8 metres per second?
Post by: Revel on October 01, 2017, 10:13:06 PM
If Earth is accelerating up at 9.8 m/s/s that means in 24 hours Earth would have accelerated by:
60s/min x 60 min/hr x 24hr/day x 9.8 m/s/s = 846,720 m/s/s per day.
Which means within 360 days the earth would have accelerated by 846,720 x 360 = 304,819,200 m/s/s.
Which means each year Earth would have accelerated beyond the speed of light. Relativity tells us this is impossible so what is the solution to this dilemma? Alternatively what is the source of energy driving the continued acceleration of Earth?

   >o< Acceleration is a measurement of the rate of change in velocity. Jerk is a measurement of acceleration's rate of change. You are treating acceleration as changing with itself: acceleration in two seconds is not 9.8m/s^2, but it sure as hell is not 19.2 m/s^2. Look: Acceleration doesn't change by its initial value, but by the value of jerk. There is a way to calculate the rate of change in acceleration, but acceleration is NOT the rate of change of acceleration, but VELOCITY. Acceleration, for all intents and purposes, remains at 9.8m/s^2, since adding jerk to the fold will complicate things to take into account the smallest margin of error. For the purpose of simplification, acceleration is constant. After two seconds, at an acceleration of 9.81 m/s^2, the VELOCITY is 19.2m/s. And by the way, the speed of light has a set value of ~299,792,458 meters per second. This is assuming that it is not influenced by any outer force, like gravity. This is assuming there is no acceleration, in the same constant pretext that you presented with the "acceleration" of light. You don't know what you're saying, user Horhang.
Title: Re: Why is the Earth accelerating at9.8 metres per second?
Post by: TomInAustin on October 02, 2017, 02:37:25 PM
a) It's not doing that (because: tides, Cavendish, equatorial-vs-poles, mountain-tops, etc).
All of which are addressed in the wiki/FAQ and in various threads on these very fora. You may disagree, or they may be flawed and you may want to debate them, but it doesn't change the fact that they exist. This thread initially started in Q&A. I know it is hard for round earth logicians to read forum descriptions, but what this thread quickly changed into is not appropriate for Q&A. It is for FE questions and answers, not for RE to point out something they disagree with. Instead of locking the thread, I moved it here (which was apparently a mistake). I suppose I can just be more strict about enforcing rules, but then many of you will complain about that, too.


b) Even if it was, then there would need to be an infinite energy source to power it
False.

Please expand on your answer.  It seems to be that in infinite power source would have to be present or we risk the energy running out and the acceleration ends.

What is the source of this energy in your opinion?

Think about it though - Energy is force times distance through which it's applied.   Force is mass times acceleration.

So we know the acceleration - and we can calculate the distance (assuming the force has been applied at about the same rate at least throughout recorded history).

The other thing is the mass.   What is the mass of the Flat Earth?   Mass is volume times density.  We know the area of the Flat Earth is infinite (Yes, I read the Wiki) and the thickness is greater than zero - and the mass is greater than zero...and therefore the force required is infinite and if the Earth moves by even one millimeter then the energy required to move it is also infinite.

Too many infinities.

Good catch, it would have to be an infinite source of power.
Not quite. UA FE doesn't have a infinite plane (from my understanding) infinite plane FE uses normal gravity. UA only applies to FE's with finite dimension, though what those are are of course unknown.

Sigh, it's very hard to keep all the delusions separated.   But I have yet to see an answer to the OP, "Why is the Earth accelerating at 9.8 metres per second?"

Title: Re: Why is the Earth accelerating at9.8 metres per second?
Post by: TomInAustin on October 02, 2017, 02:39:48 PM
* Infinite disk + UA has the "infinite energy" problem - but it's what most TFES.org people are saying.
Why must you insist on misrepresenting our position? No one is falling for it.

Becuase you don't have a collective position,  a huge reason why no one is falling for it.   But since you said that, how about you tell us what your thoughts are on the OP?  "Why is the Earth accelerating at 9.8 metres per second?"
Title: Re: Why is the Earth accelerating at9.8 metres per second?
Post by: pszemoI on October 04, 2017, 12:13:00 AM
I confess I'm a bit puzzled as to why FE'ers do this.

If the Earth is an infinite disk, of decent thickness - then regular "per Isaac Newton" gravity is a reasonable explanation.
No, it is not. Do you know Isaac Newton gravity depends on the mass of the objects?
Infinite disk would have to have infinite mass, and infinite gravity. Sounds stupid? It does.
Title: Re: Why is the Earth accelerating at9.8 metres per second?
Post by: Rama Set on October 04, 2017, 01:14:23 AM
I confess I'm a bit puzzled as to why FE'ers do this.

If the Earth is an infinite disk, of decent thickness - then regular "per Isaac Newton" gravity is a reasonable explanation.
No, it is not. Do you know Isaac Newton gravity depends on the mass of the objects?
Infinite disk would have to have infinite mass, and infinite gravity. Sounds stupid? It does.

Calculations have been done and an infinite disc made of earth like materials would have gravity approximately the same as the Earth. There is another problem that rules this out though: light would bend and reach a focal point directly above an observer where images would be highly distorted. We don't see this, ergo, the Earth is almost certainly not an infinite disc.
Title: Re: Why is the Earth accelerating at9.8 metres per second?
Post by: xenotolerance on October 04, 2017, 02:55:50 AM
Calculations have been done and an infinite disc made of earth like materials would have gravity approximately the same as the Earth. There is another problem that rules this out though: light would bend and reach a focal point directly above an observer where images would be highly distorted. We don't see this, ergo, the Earth is almost certainly not an infinite disc.

Calculations like this one (https://physics.stackexchange.com/questions/264306/gravitational-force-when-standing-on-an-infinite-disc#264311).

Quote
If the disk has infinite diameter it is nothing but an infinite plane. For any finite thickness we can consider a layer of mass whose superficial density is σσ. Moreover, if the plane is infinite it does not matter if you are one meter or one kilometer away from the plane. Wherever you look at the plane you will see the same structure. So the gravitational field cannot depend on the distance from the plane. It must be uniform and their lines must be perpendicular to the plane.
Title: Re: Why is the Earth accelerating at9.8 metres per second?
Post by: 3DGeek on October 04, 2017, 04:11:15 PM
I confess I'm a bit puzzled as to why FE'ers do this.

If the Earth is an infinite disk, of decent thickness - then regular "per Isaac Newton" gravity is a reasonable explanation.
No, it is not. Do you know Isaac Newton gravity depends on the mass of the objects?
Infinite disk would have to have infinite mass, and infinite gravity. Sounds stupid? It does.

Sorry - you're wrong.  Do this little thought experiment...suppose the FE was 1km thick - let's chop it up into 1km x 1km x 1km cubes.

Each cube produces a gravitational field on you and I.

If the FE world were finite - then in the center of the world, all of the cubes on the edges would be pulling us mostly OUTWARDS away from the center - and all of the nearby cubes would be pulling us mostly downwards and only a little bit to the sides.

But since all of the lateral pull from the cubes in front of us is precisely cancelled out by the cubes behind us - and all of the lateral pull from the ones to the left is cancelled out by the cubes to our right, the only remaining pull would be DOWNWARDS.

Suppose you now walk Northwards of the center by one kilometer.

Now there is one less row of cubes ahead of you - and one more row behind you.  So there would be a slight tug towards the South...still a decent downwards pull though - so it would seem as if you were walking downhill.   As you continue to walk North, more and more pull to the South and less and less to the North, so the perceived slope would get larger and larger.   Towards the edge of the Flat Earth, it would be more like climbing down a vertical cliff than walking on level ground.

So a FINITE FE with conventional gravity doesn't work.  BUSTED.

But if the FE is an INFINITE disk, then wherever you're standing, there are an infinite number of 1km cubes stretching off in all directions - you can walk anywhere and there are STILL an infinite number in all directions.

So the gravitational field of an infinite flat disk of finite (but large) thickness is pulling you directly downwards - just like on a round earth.

As an RE'er - this is a painful thing to have to admit - but FE (infinite world+newtonian gravity) works just fine.

I have no clue why these people insist on using Universal Acceleration - that's ridiculously easy to debunk ("Different gravity on mountain tops", "Different gravity at the equator")...where those things can be explained away (somewhat) with an infinite world with normal Newtonian gravity.

I'm pretty sure I could still debunk it - but it's much harder than UA.


Title: Re: Why is the Earth accelerating at9.8 metres per second?
Post by: Curious Squirrel on October 04, 2017, 04:24:21 PM
I confess I'm a bit puzzled as to why FE'ers do this.

If the Earth is an infinite disk, of decent thickness - then regular "per Isaac Newton" gravity is a reasonable explanation.
No, it is not. Do you know Isaac Newton gravity depends on the mass of the objects?
Infinite disk would have to have infinite mass, and infinite gravity. Sounds stupid? It does.

Sorry - you're wrong.  Do this little thought experiment...suppose the FE was 1km thick - let's chop it up into 1km x 1km x 1km cubes.

Each cube produces a gravitational field on you and I.

If the FE world were finite - then in the center of the world, all of the cubes on the edges would be pulling us mostly OUTWARDS away from the center - and all of the nearby cubes would be pulling us mostly downwards and only a little bit to the sides.

But since all of the lateral pull from the cubes in front of us is precisely cancelled out by the cubes behind us - and all of the lateral pull from the ones to the left is cancelled out by the cubes to our right, the only remaining pull would be DOWNWARDS.

Suppose you now walk Northwards of the center by one kilometer.

Now there is one less row of cubes ahead of you - and one more row behind you.  So there would be a slight tug towards the South...still a decent downwards pull though - so it would seem as if you were walking downhill.   As you continue to walk North, more and more pull to the South and less and less to the North, so the perceived slope would get larger and larger.   Towards the edge of the Flat Earth, it would be more like climbing down a vertical cliff than walking on level ground.

So a FINITE FE with conventional gravity doesn't work.  BUSTED.

But if the FE is an INFINITE disk, then wherever you're standing, there are an infinite number of 1km cubes stretching off in all directions - you can walk anywhere and there are STILL an infinite number in all directions.

So the gravitational field of an infinite flat disk of finite (but large) thickness is pulling you directly downwards - just like on a round earth.

As an RE'er - this is a painful thing to have to admit - but FE (infinite world+newtonian gravity) works just fine.

I have no clue why these people insist on using Universal Acceleration - that's ridiculously easy to debunk ("Different gravity on mountain tops", "Different gravity at the equator")...where those things can be explained away (somewhat) with an infinite world with normal Newtonian gravity.

I'm pretty sure I could still debunk it - but it's much harder than UA.
An interesting point (slightly off-topic, my apologies) is that the guys over at theflatearthsociety.org are actually much more into this whole 'infinite plane' idea than we see over here. It's one of the major differences between them that I've noticed. Although they also have a contigent that's a bit.... more extreme I suppose is a delicate way of putting it. Go look up 'denpressure' for a small taste if you're curious.